4.GASTROENTEROLOGIA Y CIRUGIA

330
GASTROENTEROLOGIA Y CIRUGIA ENARM

Transcript of 4.GASTROENTEROLOGIA Y CIRUGIA

Page 1: 4.GASTROENTEROLOGIA Y CIRUGIA

GASTROENTEROLOGIA Y CIRUGIA

ENARM

Page 2: 4.GASTROENTEROLOGIA Y CIRUGIA
Page 3: 4.GASTROENTEROLOGIA Y CIRUGIA
Page 4: 4.GASTROENTEROLOGIA Y CIRUGIA

• TGO: Marcador de NECROSIS HEPATOCELULAR.– Hígado.

– Músculo cardiaco y esquelético (el ejercicio muscular intenso provoca elevaciones de GOT)

– Riñones

– Cerebro

– Páncreas

– Células sanguíneas

• TGP: Marcador de NECROSIS HEPATOCELULAR– Más específico del hígado.

TRANSAMINASAS (GOT, GPT)

Page 5: 4.GASTROENTEROLOGIA Y CIRUGIA

NIVELES DE GOT y GPT

• Muy sensibles para detectar necrosis hepatocelular de bajo grado.

• Poca correlación con la extensión de la necrosis hepatocelular.

• No predictivo de pronóstico en hepatitis aguda.

• Pueden ser normales o casi normales en el contexto de la cirrosis hepática.

Page 6: 4.GASTROENTEROLOGIA Y CIRUGIA

FOSFATASA ALCALINA

1) ENFERMEDADES HEPATOBILIARES U ÓSEAS.1) GGT y 5’ nucleotidasa (origen hepatobiliar)

2) Valores de FA hasta 3 veces su valor normal carecen de especificidad. Puede elevarse en el contexto de cualquier hepatopatía.

3) FA> 4x Muy específico de colestasis. No diferencia intra/extrahepática, ni benigna/maligna.

4) Causas:1) Obstrucción extra hepática crónica de la VB (colangiocarcinoma,

coledocolitiasis, Ca de páncreas)2) Cirrosis biliar primaria.3) Colangitis esclerosante.4) Colestasis toxico-medicamentosa.5) Metástasis hepáticas6) Enfermedades infiltrativas7) Elevaciones fisiológicas: Embarazo 3er trimestre, crecimiento óseo y

adolescencia.

Page 7: 4.GASTROENTEROLOGIA Y CIRUGIA

BILIRRUBINA

1) Bilirrubina indirecta: No se filtra por el glomérulo porque va unida a la albúmina.1) HEMÓLISIS. Aumentos de BT<5mg/dl.2) Eritropoyesis ineficaz3) Reabsorción de hematomas.4) Defectos adquiridos o hereditarios de captación o conjugación.

El más frecuente: GILBERT.

2) Bilirrubina directa: Se filtra por el glomérulo a la orina.1) Enfermedades parenquimatosas agudas o crónicas.2) Enfermedades hepáticas con colestasis predominante (CBP, EICH,

toxicidad farmacológica…)

3) Trastornos infiltrativos difusos.4) OBSTRUCCIÓN BILIAR DE CUALQUIER ORIGEN.

Page 8: 4.GASTROENTEROLOGIA Y CIRUGIA

ACTIVIDAD DE PROTROMBINA

1) El hígado sintetiza todos los factores importantes de la coagulación EXCEPTO EL FACTOR VIII (sintetizado en endotelio vascular y células reticulo-endoteliales.1) La medición de factor VIII, diferencia un CID de una hepatopatía

severa.

2) Diagnostico diferencial alteración de AP1) Hepatopatía aguda o crónica.2) Déficit de vitamina K

1) Malabsorción2) Desnutrición3) Uso de antibióticos

3) Interferencia en la acción de la vitamina K: WARFARINA.4) Coagulopatía por consumo (por ejemplo CID).

3) Administrar vitamina K parenteral para hacer DD

Page 9: 4.GASTROENTEROLOGIA Y CIRUGIA

ALBÚMINA

1) Los hepatocitos sintetizan 10 g de albúmina/día. En la enfermedad hepática, esta síntesis disminuye.

2) Vida media de 20 días (no sirve para detectar cambios bruscos en la capacidad de síntesis).

3) Prealbúmina. También de síntesis hepática.

4) Factores extrahepáticos de disminución de albúmina:1) Desnutrición.

2) Integridad vascular.

3) Catabolismo

4) Pérdida por orina o heces.

Page 10: 4.GASTROENTEROLOGIA Y CIRUGIA

PRUEBAS DE FUNCIÓN HEPÁTICA

1) ASAT/GOT: Necrosis hepatocelular. 2) ALAT/GPT: Necrosis hepatocelular (más sensible y

específica)3) GGT: Colestasis u obstrucción biliar.4) FA: Colestasis, procesos infiltrativos, obstrucción

biliar.5) Bilirrubina: Colestasis, trastornos de la

conjugación, obstrucción biliar.6) Albúmina: Función de síntesis hepática7) AP o INR: Función de síntesis hepática8) LDH: Amplia distribución celular. Aumento masivo

en Hepatitis isquémica.

Page 11: 4.GASTROENTEROLOGIA Y CIRUGIA
Page 12: 4.GASTROENTEROLOGIA Y CIRUGIA
Page 13: 4.GASTROENTEROLOGIA Y CIRUGIA

ACALASIA• 1. Que es la acalasia? R = Es un trastorno por motilidad inadecuada que se caracteriza por una RELAJACIÓN INCOMPLETA y

AUMENTO DE LA PRESIÓN BASAL DEL EEI, además de presentar APERISTALSIS DEL CUERPO.

• 2. Que causa la acalasia? R = Es IDIOPÁTICO, aunque una de las hipótesis indica que es de origen central con DESTRUCCIÓN

PARCIAL DEL NÚCLEO DORSAL DEL VAGO o sus fibras nerviosas a nivel del esófago con la siguiente alteración de la función.

• 3. Cuales son los síntomas más comunes de la acalasia? R = DISFAGIA EN EL 90% DE LOS CASOS, INICIALMENTE A SÓLIDOS Y DESPUÉS A LÍQUIDOS. El segundo

síntoma mas frecuente es la REGURGITACIÓN EN 75% mas acentuada en decúbito, EL DOLOR 20-30% ES MENOS COMÚN, DE LOCALIZACIÓN EPIGÁSTRICA, RETROESTERNAL, E IRRADIADO HACIA CUELLO, DORSO Y EXTREMIDADES SUPERIORES.

• 4. Cuales son los estudios de gabinete necesarios para el diagnostico de acalasia? R = La MANOMETRIA ES DE ELECCIÓN la cual nos muestra una PRESIÓN DEL EEI AUMENTADA (>69

MM/HG), SIN PRESENCIA DE ONDAS PERISTÁLTICAS A LA DEGLUCIÓN. La ESOFAGOGRAFIA CON BARIO MUESTRA ALTERACIONES RELACIONADAS CON LAS ONDAS PERISTÁLTICAS Y LA AUSENCIA DE RELAJACIÓN DEL EEI DANDO UNA IMAGEN EN PICO DE PÁJARO O EN PUNTA DE LÁPIZ característica.

• 5. Cual es el manejo de la acalasia?1) MEDICO: A base de DILATACIONES HIDROSTÁTICAS O NEUMÁTICA, además de LA INYECCIÓN CON

TOXINA BOTULINICA.2) Quirúrgico: MIOTOMIA DE HELLER con una tasa de curación del 95%.

Page 14: 4.GASTROENTEROLOGIA Y CIRUGIA
Page 15: 4.GASTROENTEROLOGIA Y CIRUGIA

CASO CLINICO

A 29-year-old woman complains of dysphagiawith both solids and liquids, worse when she

is eating quickly or is anxious. Manometryreveals normal basal esophageal sphincter

pressure, with no relaxation of the sphincteron swallowing. Which of the following is themost appropriate next step in management?

(A) beta-blocker therapy(B) partial esophagectomy(C) anticholinergic drugs

(D) calcium channel blockers(E) dietary modification

Page 16: 4.GASTROENTEROLOGIA Y CIRUGIA

(D) These findings are characteristic of achalasia.

Anticholinergic medications and dietary

changes do not provide much help. Successful

therapies include nitroglycerine, nifedipine

(a calcium channel blocker), botulinum toxin

injected endoscopically, balloon dilatation, and

esophageal myotomy (not excision).

Page 17: 4.GASTROENTEROLOGIA Y CIRUGIA
Page 18: 4.GASTROENTEROLOGIA Y CIRUGIA

ESPASMO ESOFAGICO DIFUSO• 6. Que es el espasmo esofágico difuso? R = Se trata de un trastorno motor esofágico que afecta fundamentalmente a la musculatura lisa y que se caracteriza

por LA APARICIÓN DE ACTIVIDAD CONTRÁCTIL NO PROPULSIVA que alterna con episodios de peristalsis normal.

• 7. Cual es el cuadro clínico del espasmo esofágico difuso?

R = DOLOR TORAXICO CON LOCALIZACIÓN fundamentalmente RETROESTERNAL, aunque a veces SE IRRADIA A CUELLO Y A LOS HOMBROS SIMULANDO UNA ENFERMEDAD CARDIACA aumentando cuando están bajo estrés sicológico o emocional. La DISFAGIA es tan frecuente o mas que el dolor torácico.

• 8. Cuales son las alteraciones histológicas que esperas encontrar en el espasmo esofágico difuso? R = HIPERPLASIA DE LAS CÉLULAS musculares y ocasionalmente mínimas ALTERACIONES a nivel de las fibras nerviosas

terminales del PLEXO DE AUERBACH, no existiendo alteraciones significativas en las células ganglionares y nervios o en todo caso estas son mínimas. También es intermitente SE DESENCADENA CON FRECUENCIA CON LA TOMA DE SÓLIDOS O BEBIDAS FRÍAS.

• 9. Cuales son los hallazgos al gabinete en el espasmo esofágico difuso? R = ESOFAGOGRAMA: Que nos muestra hallazgos característicos con APARICIÓN DE FRECUENTES ONDAS NO

PROPULSIVAS en el musculo liso esofágico QUE INDENTAN LA COLUMNA DE BARIO y retrasan su evacuación, lo cual se describe como ESÓFAGO EN SACACORCHOS. MANOMETRIA: Alteraciones segmentarias, apareciendo generalmente en los DOS TERCIOS INFERIORES DEL CUERPO ESOFÁGICO. PHmetria obligatoria para descartar ERGE como causa del espasmo.

• 10. Cual es el manejo del espasmo esofágico difuso?

MEDICO: NIFEDIPINO, ANTIDEPRESIVOS Y ANSIOLÍTICOS.

QUIRURGICO: La ESOFAGOMIOTOMIA AMPLIADA hasta el cayado aórtico puede ser una alternativa muy acertada EN CASOS GRAVES REFRACTARIOS a cualquier otro tratamiento.

Page 19: 4.GASTROENTEROLOGIA Y CIRUGIA
Page 20: 4.GASTROENTEROLOGIA Y CIRUGIA

PERISTALSIS ESOFÁGICA SINTOMÁTICA O ESÓFAGO EN CASCANUECES

• 11. Que es la peristalsis esofágica sintomática o esófago en cascanueces?

R = Trastorno de la motilidad esofágica que se caracteriza por la PRESENCIA DE CONTRACCIONES DE GRAN AMPLITUD, SIN ALTERACIONES EN EL PERISTALTISMO del cuerpo esofágico y relajación del esfínter esofágico inferior. Se RELACIONA CON ERGE y a la histología no presenta alteraciones.

• 12. Cual es el cuadro clínico de la peristalsis esofágica sintomática o esófago en cascanueces?

R = DOLOR TORÁCICO RECURRENTE y con menor frecuencia disfagia.

• 13. Cuales son los hallazgos de gabinete en la peristalsis esofágica sintomática o esófago en cascanueces?

R = El diagnostico se establece por la MANOMETRIA y se caracteriza por la aparición de ONDAS DE GRAN AMPLITUD (180 MM/HG), QUE OCASIONALMENTE TAMBIÉN SON DE LARGA DURACIÓN (6SEG), siendo condición indispensable que EL PERISTALTISMO ESOFÁGICO SEA NORMAL.Siempre se debe descartar ERGE realizando una PHmetria.

• 14. Cual es el manejo de la peristalsis esofágica sintomática o esófago en cascanueces?

1) MEDICO: Si existe ERGE se recomiendo uso de OMEPRAZOL y evitar MIORRELAJANTES (nifedipino,

diltiazem o nitratos) POR QUE EMPEORARÍAN EL ERGE. BENZODIAZEPINAS Y SOBRE TODO LOS ANTIDEPRESIVOS TRICICLICOS parecen útiles.

2) QUIRURGICO: ESOFAGOMIOTOMIA AMPLIADA HASTA EL CAYADO AÓRTICO puede ser una alternativa en pacientes intratables. La inyección de toxina botulinica tiene resultados muy pobres.

Page 21: 4.GASTROENTEROLOGIA Y CIRUGIA
Page 22: 4.GASTROENTEROLOGIA Y CIRUGIA

EEI HIPERTENSO

• 15. Como se define el EEI hipertenso? R = Como la existencia DE UN TONO O PRESIÓN media máxima basal DEL

EEI IGUAL O SUPERIOR A DOS DEVIACIONES ESTÁNDAR, mas de 40-45 mm/Hg, siendo además condicionantes indispensables que existan RELAJACIONES COMPLETAS DEL EEI y que EL PERISTALTISMO ESOFÁGICO SEA NORMAL.

• 16. Cual es el cuadro clínico del EEI hipertenso? R = DOLOR TORÁCICO O DISFAGIA. Mejorando generalmente el dolor tras el

tratamiento, siendo la disfagia la mas difícil de tratar.

• 17. Cual es el manejo del EEI hipertenso? R = RELAJANTES DE LA MUSCULATURA LISA

Page 23: 4.GASTROENTEROLOGIA Y CIRUGIA

ENFERMEDADES SISTEMICAS ASOCIADAS A TRASTORNOS MOTORES DEL ESOFAGO

Page 24: 4.GASTROENTEROLOGIA Y CIRUGIA
Page 25: 4.GASTROENTEROLOGIA Y CIRUGIA

CASO CLINICOA 34-year-old woman complains bitterly of

heartburn. Physical examination reveals healing

lesions of the fingertips that she says were

small ulcers, and there are small areas of telangiectasias

on her face. Esophageal manometry

reveals a decrease in the expected amplitude of

smooth muscle contraction. Lower esophageal

sphincter tone is subnormal, but relaxes normally

with swallowing. Which of the following

statements regarding this condition is most

likely correct?

(A) characterized by systemic signs of inflammation

(B) predominantly treated symptomatically

(C) characterized by a poor prognosis

(D) usually more frequent in men

(E) characterized by death secondary to a renal crisis

Page 26: 4.GASTROENTEROLOGIA Y CIRUGIA

(B) These findings are characteristic of scleroderma.

If the disease is limited, the prognosis is

not necessarily poor. The limited form is characterized

by calcinosis, Raynaud’s (often with

distal ulceration), esophageal motility disorder,

sclerodactyly, and telangiectasia. It has a

female preponderance. Renal disease can be

severe, but is not the most common cause of

death. Esophageal symptoms should be treated

aggressively.

Page 27: 4.GASTROENTEROLOGIA Y CIRUGIA

ESOFAGITIS INFECCIOSA

Page 28: 4.GASTROENTEROLOGIA Y CIRUGIA

CASO CLINICO

A 27-year-old man with HIV complains of painevery time he swallows (odynophagia). He is

not on any antiretroviral therapy and otherwisefeels well. Examination of the mouth and

pharynx are normal. Which of the following isthe most likely diagnosis?

(A) midesophageal cancer(B) distal esophageal cancer

(C) peptic stricture(D) candida infection

(E) bacterial esophagitis

Page 29: 4.GASTROENTEROLOGIA Y CIRUGIA

(D) Painful swallowing can be caused by candida

or herpes infection or pill-induced esophagitis.

Patients with immunodeficiency states (e.g.,

AIDS) may have herpetic, candidal, or CMV

esophagitis, as well as tumors (lymphoma,

Kaposi sarcoma).

Page 30: 4.GASTROENTEROLOGIA Y CIRUGIA

ESOFAGITIS POR RADIACION

Page 31: 4.GASTROENTEROLOGIA Y CIRUGIA

ERGE• 18. Que es el ERGE? R = Cuando el contenido gástrico pasa a través del cardias hacia el esófago.

• 19. Cual es el cuadro clínico del ERGE? R = Triada clásica de PIROSIS, DOLOR RETROESTERNAL Y REGURGITACIONES.

• 20. Cuales son los estudios de gabinete realizados en el ERGE? R = PH METRIA 24 HORAS ES DE ELECCIÓN en todo paciente sintomático.

Endoscopia puede definir la presencia de esofagitis, displasia o metaplasia.

• 21. Cual es el manejo de los pacientes con ERGE? 1) Medidas higienicodieteticas. 2) Farmacoterapia inicial: Antiacidos antes de ingerir o acostarse y no sirven en el

tratamiento de la esofagitis. EL BETANECOL agonista colinérgico, incrementa la presión del EEI, mejora la peristalsis esofágica y aumenta el flujo salical. METOCLOPRAMIDA: antagonista de la dopamina, PROMUEVE EL VACIAMIENTO GÁSTRICO. Los antagonistas de los H2 y los IBP.

3) Manejo quirúrgico: En pacientes con recidivas y esofagitis.

Page 32: 4.GASTROENTEROLOGIA Y CIRUGIA

MANEJO QUIRURGICO ERGE

Page 33: 4.GASTROENTEROLOGIA Y CIRUGIA
Page 34: 4.GASTROENTEROLOGIA Y CIRUGIA
Page 35: 4.GASTROENTEROLOGIA Y CIRUGIA
Page 36: 4.GASTROENTEROLOGIA Y CIRUGIA

ESOFAGITIS POR CAUSTICOS

• 22. Que es la esofagitis por cáusticos? R = Entidad clínica caracterizada por la ingestión de sustancias causticas,

produciendo lesiones a nivel de la cavidad oral, esófago y estomago.

• 23. Cual es la clasificación de las lesiones en la esofagitis por cáusticos? Grado I: DAÑO A LA MUCOSA, con tejido inflamatorio abundante, edema e

hiperemia. Grado II: Ocasiona daño a la MUCOSA Y SUBMUCOSA con ulceración de los

tejidos, daño a vasos sanguíneos, con zonas de hemorragia. Grado III: Destrucción de la mucosa con PERFORACIÓN de la pared.

• 24. Que estudios de gabinete se le solicitan a un paciente con esofagitis por cáusticos?

R = Rx tórax, TAC sugiriéndose con material de contraste. La endoscopia se realiza después de descartar perforación.

Page 37: 4.GASTROENTEROLOGIA Y CIRUGIA
Page 38: 4.GASTROENTEROLOGIA Y CIRUGIA

MEMBRANAS O ANILLOS ESOFÁGICOS

• 25. Que son las membranas o anillos esofágicos? R = Son membranas que aparecen en la parte superior del esófago que pueden ser origen congénito o inflamatorio.

• 26. Cual es el cuadro clínico de las membranas o anillos esofágicos?

R = DISFAGIA INTERMITENTE PARA SÓLIDOS

• 27. Cual es el SINDROME DE PLUMMER-VINSON en las membranas o anillos esofágicos? R = Se denomina así cuando estas membranas localizadas habitualmente en la parte superior del esófago, SE ASOCIA

CON DISFAGIA EN MUJERES DE EDAD MEDIA CON ANEMIA FERROPENICA Y GLOSITIS.

• 28. En que radica la importancia del diagnostico y tratamiento oportuno de las membranas o anillos esofágicos? R = Se ha ASOCIADO CON CARCINOMA ESOFÁGICO POSCRICOIDEO que puede aparecer muchos años después de la

disfagia

• 29. Que estudios de gabinete son necesarios para el diagnostico de las membranas o anillos esofágicos? R = Estudios con bario, observándose las membranas en la proyección lateral.

• 30. Que nombre reciben las membranas o anillos esofágicos localizados en la parte inferior? R = ANILLO DE SCHATZKI O ANILLO B, que se sitúa habitualmente en la UNIÓN ESCAMOCOLUMNAR.

• 31. Cual es el manejo de las membranas o anillos esofágicos? R = DILATACIÓN en caso de disfagia.

Page 39: 4.GASTROENTEROLOGIA Y CIRUGIA
Page 40: 4.GASTROENTEROLOGIA Y CIRUGIA

DIVERTÍCULO ESOFÁGICO DE ZENKER

• 33. Donde se localiza comúnmente el divertículo esofágico de Zenker? R = Lado IZQUIERDO 50%.

• 34. Cual es el cuadro clínico del divertículo esofágico de Zenker? R = El síntoma predominante es la DISFAGIA. Además se manifiesta con

SENSACIÓN DE CUERPO EXTRAÑO en la faringe, salivación excesiva, REGURGITACIÓN DE COMIDA INGERIDA VARIOS DÍAS ANTES. Otros datos pueden ser HALITOSIS, ahogamiento y perdida de peso.

• 35. Cuales son los estudios de gabinete a realizar en el divertículo esofágico de Zenker?

R = Esofagograma y endoscopia.

• 36. Cual es el manejo del divertículo esofágico de Zenker? R = Quirúrgico con MIOTOMIA CRICOFARINGEA.

Page 41: 4.GASTROENTEROLOGIA Y CIRUGIA
Page 42: 4.GASTROENTEROLOGIA Y CIRUGIA

HERNIA HIATAL

• 37. Cuantos tipos se han identificado de hernia hiatal? 3 tipos.

1) HERNIA POR DESLIZAMIENTO: Caracterizada por un desplazamiento hacia arriba del cardias dentro del mediastino posterior.

2) HERNIA PARAESOFAGICA : Caracterizada por un desplazamiento hacia arriba del fondo gástrico a lo largo de un cardias en posición normal.

3) HERNIA COMBINADA O MIXTA: En la que hay un desplazamiento hacia arriba del cardias y el fondo gástrico.

• 38. Cual es el cuadro clínico de la hernia hiatal? R = PIROSIS Y REGURGITACIÓN que acompañan a las HERNIAS HIATALES POR

DESLIZAMIENTO. DISFAGIA Y PLENITUD POSPANDRIAL en las HERNIAS PARAESOFAGICAS

• 39. Cual es el manejo de la hernia hiatal? R = Quirúrgico.

Page 43: 4.GASTROENTEROLOGIA Y CIRUGIA
Page 44: 4.GASTROENTEROLOGIA Y CIRUGIA

CANCER DE ESOFAGO• 40. Cuales son los tipos histológicos mas frecuentes del cáncer de esófago?1) Carcinoma EPIDERMOIDE mas habitual, el cual surge a partir del EPITELIO PLANO

ESTRATIFICADO. 2) ADENOCARCINOMA el cual se aduce a cambios de metaplasia intestinal que sustituye al epitelio

plano del esófago.

• 41. Que condiciones predisponen a padecer cáncer de esófago? R = Sexo masculino, FUMADOR, alcohólico. ERGE que predispone a los cambios de metaplasia

intestinal que caracterizan al ESÓFAGO DE BARRET, lesión precursora del ADENOCARCINOMA.

• 42. Cual es el cuadro clínico de un paciente con cáncer de esófago? R = DISFAGIA, de manera menos frecuente dolor retroesternal al deglutir y presencia de síntomas

respiratorios como como tos y expectoración.

• 43. Cuales son los estudios de gabinete realizados para diagnostico del cáncer de esófago? R = ESOFAGOGRAMA al observar un estrechamiento en la luz del esófago, con perdida del modelo

mucoso, escasa distensibilidad del segmento afectado y dilatación preestenotica. TAC Y US PARA VALORAR EXTENSIÓN.

• 44. Cual es el manejo del cáncer de esófago? R = Quimio, radio o cirugía lo cual depende del estadio de la enfermedad.

Page 45: 4.GASTROENTEROLOGIA Y CIRUGIA
Page 46: 4.GASTROENTEROLOGIA Y CIRUGIA
Page 47: 4.GASTROENTEROLOGIA Y CIRUGIA

CASO CLINICO

A 71-year-old man develops progressive weight loss and dysphagia over a 3-month period. Upper endoscopy and biopsy of the lesion confirm esophageal cancer. Which of the following statements regarding this cancer is most likely correct?

(A) is very responsive to chemotherapy

(B) is more common in females

(C) has a 5-year cure rate of 20%

(D) may be either adenocarcinoma or squamous celL carcinoma

(E) is characterized by significant complications from hemorrhage

Page 48: 4.GASTROENTEROLOGIA Y CIRUGIA

(D) This presentation is typical of esophagealcancer. Lesions in the upper two-thirds of theesophagus are squamous, but in the distalesophagus, most are adenocarcinomas. Theadenocarcinomas develop more commonlyfrom columnar epithelium in the distal esophagus(Barrett’s esophagus). Adenocarcinomasof the esophagus have the biologic behavior ofgastric cancers. The incidence of squamous cellcancer of the esophagus is decreasing whileadenocarcinoma is increasing. Currently, over50% of esophageal cancer is adenocarcinoma.The 5-year survival for esophageal cancer is<5%. Combination therapy seems to be moreeffective than surgery alone.

Page 49: 4.GASTROENTEROLOGIA Y CIRUGIA

ENFERMEDAD ACIDO PÉPTICA• 45. Cuales son los factores de riesgo para enfermedad acido péptica? R = Infección por HELICOBACTER PYLORI y el uso de AINES.

• 46. Cual es el cuadro clínico de un paciente que padece enfermedad acido péptica? R = DOLOR EPIGÁSTRICO, el cual se puede referir por el paciente como dispepsia. El dolor producido por las

ulceras duodenales PUEDE DESAPARECER CON EL CONSUMO DE ALIMENTOS, LECHE, O CON ANTIÁCIDOS PERO VUELVE.

• 47. Cual es la definición de dispepsia? R = Se define como DOLOR CRÓNICO O RECURRENTE o malestar localizado en el ABDOMEN SUPERIOR. Se

puede asociar con otros síntomas como nausea, saciedad, eructos y perdida de peso

• 48. Cuales son las complicaciones de la enfermedad acido péptica? R = Incluye HTDA o HTDB, perforación y obstrucción del tracto de salida del estomago.

• 49. Como diagnosticas la enfermedad acido péptica?

R = PANENDOSCOPIA DE ELECCIÓN. Si se desea encontrar H. PYLORI se puede realizar una endoscopia con biopsia de la mucosa gástrica del antro o cuerpo, CULTIVO O LA PRUEBA RÁPIDA DE UREASA. La PRUEBA DE ALIENTO DE UREA y la prueba de DETECCIÓN DE ANTÍGENO FECAL.

• 50. Cual es el manejo de la enfermedad acido péptica? R = Se divide en 3 escenarios dependiendo de la etiología. 1 ) Tx de enfermedad acido péptica por H. Pylori:

OMEPRAZOL 20 mg/12 + CLARITROMICINA 500/12 + AMOXICILINA 1gr/12, en caso de alérgicos a la penicilina se utiliza el METRONIDAZOL. 2 ) Tx de la enfermedad acido péptica relacionada con AINES: SUSPENDERLOS, en caso de no poder se recomiendo ibuprofeno, misoprostol, omeprazol o antagonistas H2 y COX-2. 3 ) Tx de la enfermedad acido péptica idiopática: Son los px que su enfermedad no es causada por el motivo 1 y 2 antes mencionado, requiere una mayor dosificación de inhibidores de bomba y la enfermedad es recidivante.

Page 50: 4.GASTROENTEROLOGIA Y CIRUGIA
Page 51: 4.GASTROENTEROLOGIA Y CIRUGIA
Page 52: 4.GASTROENTEROLOGIA Y CIRUGIA
Page 53: 4.GASTROENTEROLOGIA Y CIRUGIA

CASO CLINICO

A 54-year-old man complains of burning epigastricpain that usually improves after a meal,

and is occasionally relieved with antacids. Onexamination, he appears well and besides someepigastric tenderness on palpation, the rest ofthe examination is normal. Upper endoscopy

confirms a duodenal ulcer. Which of the followingstatements concerning PUD is most

likely correct?

(A) duodenal ulcer is seen more often in older people than is gastric ulcer(B) clinically, gastric ulcers are more common than duodenal ulcers

(C) duodenal ulcers can frequently be malignant(D) infection can cause both types of peptic ulcer

(E) peptic gastric ulcers are usually quite proximal in the stomach

Page 54: 4.GASTROENTEROLOGIA Y CIRUGIA

(D) Helicobacter pylori infection is the cause ofmost peptic ulcers, and the usual route of infectionis via the water supply. Duodenal ulcer isclinically more common, although the prevalenceon autopsy series is similar. Duodenalulcer does not represent a malignant potential.Gastric ulcers are seen in an older populationand can be malignant. Benign peptic gastric ulcerstend to be more distal.

Page 55: 4.GASTROENTEROLOGIA Y CIRUGIA

CASO CLINICO

A 73-year-old woman presents to the emergencyroom with black tarry stools and symptoms

of presyncope when standing up. Digitalrectal examination confirms the presence ofmelena. She recently started using ibuprofen

for hip discomfort. Upper endoscopy confirmsthe diagnosis of a gastric ulcer. Which of the following

is the most likely explanation for thegastric ulcer?

(A) increasing acid production(B) causing direct epithelial cell death

(C) promoting replication of Helicobacter pylori(D) an antiplatelet effect

(E) inhibiting mucosal repair

Page 56: 4.GASTROENTEROLOGIA Y CIRUGIA

(E) NSAIDs inhibit prostaglandins, which play

an important role in maintaining gastroduodenal

mucosal integrity and repair

Page 57: 4.GASTROENTEROLOGIA Y CIRUGIA

GASTRITIS AUTOINMUNE

Page 58: 4.GASTROENTEROLOGIA Y CIRUGIA

CASO CLINICOA 57-year-old woman experiences frequent

symptoms of heartburn. She reports no weight

loss, diarrhea, or dysphagia symptoms. Past

medical history is significant for B12 deficiency

treated with monthly injections. She appears

well, and the examination is normal, except for

some patches on her arms where she has lost all

the skin pigmentation. Her upper endoscopy is

normal, except for gastric biopsies commenting

on gastritis. (SELECT ONE)

(A) bile acid reflux implicated in pathogenesis

(B) immune or autoimmune pathogenesis suspected

(C) ischemia of the gastric mucosa implicated in the pathogenesis

(D) associated with Helicobacter pylori infection

(E) unknown mechanism

(F) interruption of prostaglandin synthesis

Page 59: 4.GASTROENTEROLOGIA Y CIRUGIA

(B) Type A chronic gastritis may lead to pernicious

anemia. Antibodies to parietal cells and to

intrinsic factor are frequently seen in the sera,

suggesting an immune or autoimmune pathogenesis.

These patients can also have autoimmune

thyroid disease and vitiligo.

Page 60: 4.GASTROENTEROLOGIA Y CIRUGIA

GASTRITIS ASOCIADA A H. PILORY

Page 61: 4.GASTROENTEROLOGIA Y CIRUGIA

SX DE ZOLLINGER-ELLISON

Page 62: 4.GASTROENTEROLOGIA Y CIRUGIA

TX MEDICO/ GENERALIDADES UGD

Page 63: 4.GASTROENTEROLOGIA Y CIRUGIA

TA QX UGD

Page 64: 4.GASTROENTEROLOGIA Y CIRUGIA
Page 65: 4.GASTROENTEROLOGIA Y CIRUGIA

SX DUMPING

Page 66: 4.GASTROENTEROLOGIA Y CIRUGIA

COMPLICACIONES SISTEMICAS POSTGASTRECTOMIA

Page 67: 4.GASTROENTEROLOGIA Y CIRUGIA

ENFERMEDAD DE MENETRIER

• 51. Que caracteriza a la enfermedad de Menetrier? R = Los PLIEGUES GÁSTRICOS se encuentran AUMENTADOS DE TAMAÑO

debido a una hiperplasia foveolar difusa, edema y cierto grado de inflamación.

• 52. Cual es el cuadro clínico de la enfermedad de Menetrier? R = Los pacientes se presentan con PERDIDA DE PESO, DOLOR EPIGÁSTRICO,

vomito, anorexia, dispepsia, HEMATEMESIS y SANGRE OCULTA EN HECES.

• 53. Con que enfermedad se relaciona la enfermedad de Menetrier? R = Se asocia típicamente con la GASTROPATÍA PERDEDORA DE PROTEÍNAS

y con HIPOCLORIDIA

• 54. Que es la gastropatía hiperplasica hipersecretora? R = Es una condición poco común que se caracteriza POR PLIEGUES

GÁSTRICOS GIGANTES asociados con HIPERPLASIA EPITELIAL

Page 68: 4.GASTROENTEROLOGIA Y CIRUGIA
Page 69: 4.GASTROENTEROLOGIA Y CIRUGIA

CASO CLINICO

A 59-year-old man presents with abdominalpain, anorexia, and nausea. He has lost

weight and started to develop pedal edema.Endoscopy reveals large gastric mucosal

folds. Which of the following explanationsfor the edema is most likely correct?

(A) hypoalbuminemia secondary to malnutrition(B) hypoalbuminemia secondary to protein loss

(C) impaired hepatic synthesis of albumin(D) humorally mediated cardiac disease

(E) constrictive pericarditis

Page 70: 4.GASTROENTEROLOGIA Y CIRUGIA

(B) The clinical description suggests Ménétrier’s

disease; however, biopsy is essential to rule out

lymphoma or carcinoma. The edema is usually

secondary to protein-losing enteropathy.

Treatment consists of a high protein diet, anticholinergic

therapy, and H2-blockers. In some

cases, gastrectomy is required.

Page 71: 4.GASTROENTEROLOGIA Y CIRUGIA

CÁNCER GÁSTRICO• 55. Cual es la estirpe etiológica mas común del cáncer gástrico? R = ADENOCARCINOMA.

• 56. Cual es el cuadro clínico de un paciente con cáncer gástrico? R = PERDIDA DE PESO, dolor abdominal, SACIEDAD TEMPRANA. Generalmente se presentan

con palidez. Entre las manifestaciones dermatológicas mas frecuentes se encuentran la ACANTOSIS NIGRICANS y SIGNO DE LESER-TRELAT (queratosis seborreica de aparición súbita)

• 57. Como se manifiesta la metastasis comúnmente el cáncer gástrico? R = GANGLIOS EN SUPRACLAVICULAR (Virchow), GANGLIOS PERIUMBILICALES (Hermana

Maria Jose), estructuras rectales (signo del anaquel de blúmer), OVARIOS (tumor de Krukenberg).

• 58. Cual es el estándar de oro para el diagnostico del cáncer gástrico?

R = PANENDOSCOPIA con toma de 7 BIOPSIAS de la lesión y después TAC PARA ESTADIFICACION en caso de metástasis.

• 59. Cual es el manejo del cáncer gástrico? R = Quirúrgico. La gastrectomía total se recomienda en caso de lesiones en el estomago

proximal. La QUIMIOTERAPIA no mejora la sobrevida sobre la cirugía, sin embargo COMBINADA CON LA RADIOTERAPIA si la mejora.

Page 72: 4.GASTROENTEROLOGIA Y CIRUGIA

LINFOMA GASTRICO

• 60. Cual es la estirpe etiológica mas común en el linfoma gástrico?

R = No HODKING 95%

• 61. Cual es el cuadro clínico del linfoma gástrico? R = Sintomatología no especifica como DOLOR ABDOMINAL, nausea,

vomito, PERDIDA DE PESO, SUDORES NOCTURNOS y diarrea.

• 62. Cual es la apariencia del linfoma gástrico a la endoscopia? R = Aparecen como LESIONES FUNGOIDES, PÓLIPOS, ENGROSAMIENTO DE

PLIEGUES.

• 63. Cual es el manejo del linfoma gástrico? R = Quirúrgico

Page 73: 4.GASTROENTEROLOGIA Y CIRUGIA

MESOTELIOMA

Page 74: 4.GASTROENTEROLOGIA Y CIRUGIA

CASO CLINICO

A 56-year-old woman presents with weight lost and watery diarrhea. She also notices periumbilical pain and bloating after eating. A small bowel x-ray reveals an area of narrowing and the lesion is surgically resected. The pathology report shows this to be the most common endocrine tumor of the GI tract. (SELECT ONE)

(A) polypoid adenoma

(B) leiomyoma

(C) lipoma

(D) adenocarcinoma

(E) primary gastrointestinal (GI) lymphoma

(F) carcinoid tumor

Page 75: 4.GASTROENTEROLOGIA Y CIRUGIA

(F) Carcinoid tumors account for up to 75% of all GIendocrine tumors. They are frequently multiple.Primary carcinoid tumors of the appendix are commonbut rarely metastasize. Those in the large colon maymetastasize but do not function. Carcinoids are themost common GI endocrine tumors. They arise fromneuroendocrine cells most commonly in the GI tract,pancreas, or bronchi. GI carcinoids cause abdominalpain, bleeding, or even obstruction (usually viaintussusception). Carcinoid syndrome is characterizedby flushing, diarrhea, and valvular heart disease.

Page 76: 4.GASTROENTEROLOGIA Y CIRUGIA

CASO CLINICO

A 72-year-old man is investigated for iron deficiency anemia. There is no clinical history of upper or lower GI bleeding, but his stools are fecal occult blood positive. His colonoscopy is negative for any obvious sources of bleeding, but upper endoscopy reveals ulceration in the distal duodenum that is biopsied. The pathology report shows this to be the most common primary malignancy of the small bowel. (SELECT ONE)

(A) polypoid adenoma

(B) leiomyoma

(C) lipoma

(D) adenocarcinoma

(E) primary gastrointestinal (GI) lymphoma

(F) carcinoid tumor

Page 77: 4.GASTROENTEROLOGIA Y CIRUGIA

(D) Adenocarcinoma of the small bowel

accounts for about 50% of malignant tumors of

the small bowel. They are most commonly

found in the distal duodenum and proximal

jejunum. Hemorrhage, or obstruction, is the

most common presenting symptom. X-ray findings

can mimic chronic duodenal ulcer disease

or Crohn’s disease.

Page 78: 4.GASTROENTEROLOGIA Y CIRUGIA

CASO CLINICO

A 23-year-old man of Middle Eastern decent in investigated for symptoms of chronic diarrhea vomiting, and abdominal cramps. He has lost 20 lb and appears unwell. Asmall bowel x-ray shows diffuse nonspecific mucosal abnormality. Upper endoscopy is performed to visualize the small bowel and obtain biopsies. The pathology report identifies the mucosal abnormality as “immunoproliferative small intestinal disease” (IPSID). This form of small bowel tumor can be treated with antibiotics in its early stages.

(A) polypoid adenoma

(B) leiomyoma

(C) lipoma

(D) adenocarcinoma

(E) primary gastrointestinal (GI) lymphoma

(F) carcinoid tumor

Page 79: 4.GASTROENTEROLOGIA Y CIRUGIA

(E) There is one unique form of lymphoma called

IPSID or Mediterranean lymphoma. It diffusely

involves the small intestine and usually presents

with diarrhea and steatorrhea. Oral antibiotics,

early in the disease, provide some benefit, suggesting

a possible infectious component to the

disorder. Antibiotics and chemotherapy are frequently

combined.

Page 80: 4.GASTROENTEROLOGIA Y CIRUGIA

CASO CLINICO

A 21-year-old woman develops acute periumbilical pain that localizes to the right lower quadrant. She appears unwell, and on examination, there is tenderness and guarding in the right lower quadrant. Apregnancy test is negative and a pelvic examination rules out pelvic inflammatory disease. A clinical diagnosis of acute appendicitis is made and she has an uncomplicated laparoscopic appendectomy. The pathology report notes acute inflammation and a tumor for which the appendix is a very common site of involvement. (SELECT ONE)

(A) polypoid adenoma

(B) leiomyoma

(C) lipoma

(D) adenocarcinoma

(E) primary gastrointestinal (GI) lymphoma

(F) carcinoid tumor

Page 81: 4.GASTROENTEROLOGIA Y CIRUGIA

(F) Appendiceal tumors make up nearly half of

all carcinoid tumors and are a frequent finding

in routine appendectomy specimens. They are

usually small, solitary, and benign. Even if they

invade locally, they rarely metastasize.

Page 82: 4.GASTROENTEROLOGIA Y CIRUGIA

CASO CLINICO

A 57-year-old woman with a 20-year history of celiac disease now presents with weight loss, cramps, and abdominal discomfort. She is adherent to her gluten-free diet, and is not taking any new medications. On examination, her abdomen is soft and there is fullness in the right lower quadrant. A small bowel x-ray reveals an area of narrowing in the distal ileum and there is thickening and nodularity of the mucosal folds. She undergoes surgical resection of the lesion, and the pathology report confirms a tumor that is more common in patients with celiac disease, Crohn’s disease, or depressed immune function such as human immunodeficiency virus (HIV).

(A) polypoid adenoma

(B) leiomyoma

(C) lipoma

(D) adenocarcinoma

(E) primary gastrointestinal (GI) lymphoma

(F) carcinoid tumor

Page 83: 4.GASTROENTEROLOGIA Y CIRUGIA

(E) Primary small bowel lymphoma is more

common in the settings of celiac disease,

regional enteritis, congenital immune disorders,

prior organ transplantation, autoimmune

disorders, and AIDS.

Page 84: 4.GASTROENTEROLOGIA Y CIRUGIA

CASO CLINICO

A 48-year-old man presentswith periumbilical painmade worse by eating andweight loss. A small bowelx-ray reveals an area ofnarrowing in the distalileum. The differentialdiagnosis includes tumorsthat most commonly involvethe distal part of the smallbowel. (SELECT TWO)

(A) polypoid adenoma

(B) leiomyoma

(C) lipoma

(D) adenocarcinoma

(E) primary gastrointestinal (GI) lymphoma

(F) carcinoid tumor

Page 85: 4.GASTROENTEROLOGIA Y CIRUGIA

(C, F) Lipomas and carcinoid tumors are mostfrequently found in the distal ileum.Adenocarcinomas are characteristically moreproximal. The ileum has more lymphatic tissue,than the rest of the small bowel, so lymphomais more common here than in the jejunum orduodenum. However, there is not as strong apredilection for the distal ileum as there is forlipomas and carcinoids.

Page 86: 4.GASTROENTEROLOGIA Y CIRUGIA

DIARREA AGUDA• 64. Cuales son las bacterias productoras de enterotoxinas? R = V. CHOLERAE (heces en agua de arroz), CLOSTRIDIUM PERFRINGES y E. COLI ENTEROTOXIGENA (diarrea del

viajero).

• 65. Cual es el agente causal de la colitis seudomembranosa? R = CLOSTRIDIUM DIFFICILE (exotoxina termolábil A entorotoxica y B citotoxica) es el principal agente

etiológico de las DIARREAS ASOCIADAS A ANTIBIÓTICOS.

• 66. Cual es el cuadro clínico de la colitis seudomembranosa? R = FIEBRE, nauseas, DISTENCIÓN y DOLOR ABDOMINAL, DIARREA y deshidratación.

• 67. Cual es el hallazgo de laboratorio en la colitis seudomembranosa? R = LEUCOCITOSIS, hipoalbuminemia y alteraciones electrolíticas

• 68. Cual es el hallazgo a la endoscopia en la colitis seudomembranosa? R = SEUDOMEMBRANAS

• 69. Cual es el fármaco de elección en la colitis seudomembranosa?

R = METRONIDAZOL x 10 días

• 70. Cual es el fármaco de elección colitis seudomembranosa pero en pacientes EMBARAZADAS, NIÑOS MENORES 10ª e INMUNOSUPRIMIDOS?

R = VANCOMICINA x 10 días

Page 87: 4.GASTROENTEROLOGIA Y CIRUGIA

• 71. Cual es el agente causal relacionado con diarrea y Síndrome de Guillan-Barre? R = CAMPYLOBACTER

• 72. Que cepa del E.Coli se relaciona con síndrome hemolítico urémico? R = E. COLI ENTEROHEMORRAGICA 0157:H7

• 73. Cual es el fármaco de elección para la diarrea del viajero? R = QUINOLONAS, Rifaximina nuevo fármaco que también puede utilizarse en caso de

resistencia a las quinolonas.

• 74. Cual es el agente causal del cólera? R = VIBRIO CHOLERAE, GRAM -, productor de ENTEROTOXINA, la SUBUNIDAD A de la toxina

activa de manera irreversible a la ADENILCICLASA de la mucosa intestinal, produciendo secreción de líquidos y electrolitos.

• 75. Cuales son los factores de riesgo para adquirir cólera? R = Ingesta de MARISCOS, agua contaminada.

• 76. Cual es el cuadro clínico del cólera? R = Diarrea en AGUA DE ARROZ, vomito, letargo, alteraciones de la conciencia y

convulsiones.

• 77. Cual es el manejo del cólera?

R = Hidratación y TETRACICLINA.

Page 88: 4.GASTROENTEROLOGIA Y CIRUGIA
Page 89: 4.GASTROENTEROLOGIA Y CIRUGIA
Page 90: 4.GASTROENTEROLOGIA Y CIRUGIA

DIARREA CRONICA

• 78. De acuerdo a su fisiopatología en que modalidades se divide la diarrea crónica? R = Diarrea osmótica, diarrea secretora, diarrea inflamatoria, diarrea por alteración de la motilidad y diarrea

facticia.

• 79. Que causa la diarrea osmótica y que la corrige? R = Es causada por la INGESTIÓN DE SUSTANCIAS que se absorben de manera inadecuada como el manitol,

sorbitol, LAXANTES o LACTULOSA y se CORRIGE MEDIANTE EL AYUNO.

• 80. Que causa la diarrea secretora, cual es su cuadro clínico y como se corrige? R = Es causada por ENTEROTOXINAS, TUMORES SECRETORES DE HORMONAS o adenoma velloso. Se caracteriza

por producir HECES ACUOSAS Y EN GRAN VOLUMEN y a diferencia de la diarrea osmótica esta NO SE CORRIGE CON EL AYUNO.

• 81. Que causa la diarrea inflamatoria? R = CUCI, CHRON, radioterapia

• 82. Que causa la diarrea por alteración en la motilidad intestinal? R = Colon irritable, diarrea postvagotomia, neutropenia diabética o por el hipertiroidismo.

• 83. Que causa la diarrea facticia? R = Abuso de laxantes.

Page 91: 4.GASTROENTEROLOGIA Y CIRUGIA
Page 92: 4.GASTROENTEROLOGIA Y CIRUGIA
Page 93: 4.GASTROENTEROLOGIA Y CIRUGIA

CASO CLINICO

A 30-year-old man complains of chronic diarrhea

for the past 6 months. There is no weight

loss, fever, or abdominal pain. He takes no medications

and feels fine. His physical examination

is completely normal. Further history reveals

that this man does not take laxatives; however,

in an effort to stay slim he eats a lot of sugar-freegum and sugarless candy.

Which of the following explanations is the most likely cause of his

diarrhea?

(A) direct stimulant effect of chemicals in the candies

(B) lack of fiber in his diet

(C) pancreatic insufficiency secondary to chronic protein-calorie malnutrition

(D) secondary intestinal mucosal atrophy

(E) nonabsorbed carbohydrates

Page 94: 4.GASTROENTEROLOGIA Y CIRUGIA

(E) Sugarless gums and candy often contain

sorbitol, a sugar that is not absorbed in the gut.

It thus produces an osmotic diarrhea if present

in sufficient quantity.

Page 95: 4.GASTROENTEROLOGIA Y CIRUGIA

MALABSORCION• 84. Cuales son los datos clínicos de malabsorción? R = Diarrea, DISMINUCIÓN DE PESO y desnutrición, acompañados de

DISTENCIÓN ABDOMINAL, síntomas y signos secundaros a los déficit específicos, ESTEATORREA.

• 85. Que datos de laboratorio se encuentran en un paciente con malabsorción?

R = Hipocolesterolemia, HIPOALBUMINEMIA, alteraciones electrolíticas y PH.

• 86. Cual es la posología de la prueba con D-Xilosa en malabsorción? R = Se da EN AYUNAS midiendo su nivel en suero 1hr después y la excreción en

orina en 6hr, este azúcar SE ABSORBE EN FORMA PASIVA SI LA MUCOSA INTESTINAL ESTA INTEGRA.

• 88. Cuales son las principales causas de malabsorción? R = DEFICIENCIA DE LACTASA, ESPRUE celiaco, ESPRUE TROPICAL,

ENFERMEDAD DE WIPPLE, sobre crecimiento bacteriano y síndrome de intestino corto.

Page 96: 4.GASTROENTEROLOGIA Y CIRUGIA
Page 97: 4.GASTROENTEROLOGIA Y CIRUGIA
Page 98: 4.GASTROENTEROLOGIA Y CIRUGIA

CASO CLINICO

A 23-year-old woman presents with weight loss and chronic diarrhea. She appears unwell and cachectic. Routine laboratory tests reveal a low hemoglobin level and an increased international normalized ratio (INR) even though she is not taking any anticoagulants. The liver enzymes are normal, but the albumin and calcium levels are low, suggesting generalized malnutrition. Which of the following is the most appropriate initial diagnostic test for malabsorption?

(A) xylose absorption(B) Schilling test(C) x-ray studies(D) stool fat quantitation(E) small intestinal biopsy

Page 99: 4.GASTROENTEROLOGIA Y CIRUGIA

(D) Fat malabsorption demonstrated on stool

collection for 72 hours is the gold standard, but

does not indicate the exact cause. The Schilling

test is useful in testing for vitamin B12 absorption.

X-rays can be helpful in diagnosing

underlying disorders, but are nonspecific.

Small intestinal biopsy is useful in determining

the cause of malabsorption.

Page 100: 4.GASTROENTEROLOGIA Y CIRUGIA

CASO CLINICO

A 52-year-old man has suffered with chronicdiarrhea for several years, but has refused tosee a doctor. He finally comes because he ishaving trouble driving at night, because of

difficulty seeing. Physical examination revealsa slender, pale, unwell-looking man. He has amicrocytic anemia, low calcium, and albumin

levels. Which of the following Is the mostlikely cause for his diarrhea?

(A) malabsorption(B) osmotic diarrhea

(C) secretory diarrhea(D) inflammatory bowel disease

(E) colonic tumor

Page 101: 4.GASTROENTEROLOGIA Y CIRUGIA

(A) His anemia and general appearance are

compatible with chronic inflammatory bowel

disease as well, but his night blindness suggests

vitamin Adeficiency, which is much more likely

in a malabsorption syndrome.

Page 102: 4.GASTROENTEROLOGIA Y CIRUGIA

ESPRUE CELIACO• 89. Que es el esprue celiaco? R = Causa malabsorción debido a lesión ocasionada por el GLUTEN a nivel intestinal, el cual

es contenido en el trigo, CEBADA, CENTENO Y AVENA.

• 90. Cual es el cuadro clínico del esprue celiaco? R = Hay malabsorción de CHBTS, PROTEÍNAS Y GRASAS. Perdida progresiva de peso,

distensión abdominal, flatulencia y diarrea, evacuaciones voluminosas y de mal olor.

• 91. Cuales estudios de diagnostico hay para el esprue celiaco y cuales son los anticuerpos especificos?

R = La biopsia en LA UNIÓN DUODENOYUYENAL que suele REPORTAR ATROFIA O ACORTAMIENTO DE LAS VELLOSIDADES, HIPERPLASIA DE LAS CRIPTAS y aumento en la mitosis CON INCREMENTO DE LINFOCITOS, NEUTROFILOS, MASTOCITOS Y EOSINOFILOS EN LAMINA PROPIA. Cuantificación de grasa en heces de 24 hrs mayor a 7g/dia , la ABSORCIÓN DE D-XILOSA SUELE SER ANORMAL, los ANTICUERPOS SÉRICOS ANTIGLIADINA SON

POSITIVOS EN 90%, el anticuerpo IgA es el mas especifico. Los anticuerpos IgAANTIENDOMISIO Y ANTIRRETICULINA son los MAS ESPECÍFICOS.

• 92. Cual es el manejo del esprue celiaco? R = Dieta libre de gluten

Page 103: 4.GASTROENTEROLOGIA Y CIRUGIA
Page 104: 4.GASTROENTEROLOGIA Y CIRUGIA
Page 105: 4.GASTROENTEROLOGIA Y CIRUGIA

ESPRUE TROPICAL

• 93. Cual es la etiología del esprue tropical? R = Es producido por la COLONIZACIÓN PERSISTENTE del intestino delgado por KLEBSIELLA

PNEUMONIAE, ENTEROBACTER CLOACAE o E. COLI.

• 94. Cual es el cuadro clínico del esprue tropical? R = CRISIS DE DIARREA ACUOSA, acompañada de DOLOR ABDOMINAL y FLATULENCIA, los

cuales disminuyen de intensidad a la semana de evolución y posteriormente se hacen crónicos.

• 95. Cuales son los estudios diagnósticos del esprue tropical? R = La biopsia de intestino delgado reporta ALARGAMIENTO DE LAS CRIPTAS,

ensanchamiento y acortamiento de las vellosidades e INFILTRACIÓN DE LA LAMINA PROPIA POR CÉLULAS INFLAMATORIAS. Se encuentra esteatorrea, ABSORCIÓN DE D-XILOSA ANORMAL.

• 96. Cual es el manejo del esprue tropical?

R = TETRACICLINA y ACIDO FÓLICO para mejorar ANEMIA MEGALOBLASTICA.

Page 106: 4.GASTROENTEROLOGIA Y CIRUGIA

ENFERMEDAD DE WHIPPLE• 97. Cual es el agente causal de la enfermedad de Whipple? R = Es secundaria a la infección por TROPHERYMA WHIPELLII y se relaciona con HLA B27 + en 25%.

• 98. Cual es el cuadro clínico de la enfermedad de Whipple?

R = Perdida de peso, diarrea o esteatorrea, artralgias, FIEBRE, dolor abdominal y HEMORRAGIAINTESTINAL, LAS ARTRALGIAS SON MIGRATORIAS, no destructivas y afectan las grandes

articulaciones, se presenta SACROILEITIS EN 25%. Un signo característico es la MIORRITMIA OCULOMASTICATORIA, la cual consiste en la convergencia rítmica de los ojos asociada a contracciones sincrónicas de los músculos de la masticación. A nivel ocular se manifiesta como

UVEÍTIS, vitreitis, retinitis y papiledema. A la ex fis se encuentra LINFADENOPATIAperiférica.

• 99. Cuales son los hallazgos de laboratorio en la enfermedad de Whipple?

R = La biopsia de intestino delgado muestra MACRÓFAGOS POSITIVOS CON LA TINCIÓN PAS. Sin embargo se debe demostrar el AGENTE CAUSAL EN EL LCR, tejido cerebral o GANGLIOS LINFÁTICOS. Anemia secundaria a enfermedad crónica, deficiencia de hierro, folato o VB12, asociada a neutrofilia.

Prueba de D-Xilosa anormal, LINFADENOPATIA ABDOMINAL detectada por ultrasonido. Radiografía de TORAX CON INFILTRADOS, FIBROSIS O DERRAME PLEURAL.

• 100. Cual es el manejo de la enfermedad de Whipple?

R = TMP/SMZ durante 1ª, si hay ALERGIA A SULFAMIDAS esta indicada la PENICILINA por vía oral durante el mismo periodo y se deben tratar los trastornos nutricios.

Page 107: 4.GASTROENTEROLOGIA Y CIRUGIA
Page 108: 4.GASTROENTEROLOGIA Y CIRUGIA

ILEO PARALITICO• 101. Que puede ocasionar un íleo paralitico? R = Causado por procedimientos cx, medicamentos , trastornos

endocrinos (tiroideos, paratiroideos o diabetes) y metabólicos.

• 102. Cual es el cuadro clínico en un íleo paralitico? R = DISTENCIÓN ABDOMINAL difusa o localizada en el sitio de

inflamación, con timpanismo, RUIDOS INTESTINALES HIPOACTIVOS O AUSENTES dolor mínimo.

• 103. Cual es el dato radiológico del íleo paralitico? R = Rx abdomen con NIVELES HIDROAEREOS y ASA EN

CENTINELA.

• 104. Cual es el manejo del íleo paralitico? R = Por lo general es autodelimitado.

Page 109: 4.GASTROENTEROLOGIA Y CIRUGIA
Page 110: 4.GASTROENTEROLOGIA Y CIRUGIA

SX COLON IRRITABLE• 105. Cuales son los criterios de ROMA II para integrar el síndrome de colon irritable? R = Que los síntomas hallan durado POR LO MENOS 12 SEMANAS (no necesariamente

continuas) durante los 12m previos de malestar o dolor abdominal que tiene 2 de las siguientes características. Se dice que el síndrome de intestino irritable se diagnostica cuando cumple al menos con 1 o mas de los criterios 2, 4 y 6 o bien 1, 3 y 5 DEPENDIENDO DE SI ES DIARREA LA QUE PREDOMINA O CONSTIPACIÓN. Se exacerba con el estrés y es MAS FRECUENTE EN MUJERES.

1. Menos de 3 evacuaciones por semana2. Mas de 3 evacuaciones al día3. Evacuaciones de consistencia dura4. Evacuaciones sueltas o duras5. Pujo al evacuar6. Urgencia al evacuar7. Sensación de evacuación completa8. Moco durante la evacuación9. Distención abdominal

• 106. Cual es el manejo del síndrome de colon irritable? R = Debe ser INDIVIDUALIZADO a los síntomas. PInaverio con dimeticona.

Page 111: 4.GASTROENTEROLOGIA Y CIRUGIA
Page 112: 4.GASTROENTEROLOGIA Y CIRUGIA

ENFERMEDAD DIVERTICULAR

• 107. Que causa la diverticulitis? R = Es causada por inflamación o PERFORACIÓN de alguno de los divertículos con ABSCESO

PERICOLONICO.

• 108. Cual es el cuadro clínico de la diverticulitis? R = DOLOR EN FII (apendicitis izquierda) y se puede irradiar al área suprapúbica, ingle o

espalda. Fiebre y puede cursar con ILEO, DISTENCIÓN Y VOMITO.

• 109. Que encontraras en los exámenes de laboratorio o gabinete en pacientes con diverticulitis?

R = LEUCOCITOSIS con predominio de segmentados, disuria, neumaturia, piuria o fecaluriaen caso de FISTULA. Para confirmar el diagnostico se indica una TAC. Debe EVITARSE EL ENEMA DE CONTRASTE BARITADO.

• 110. Cual es el manejo de la diverticulitis?1) Intrahospitalariamente CEFALOSPORINA Y METRONIDAZOL. 2) En casos repetidos se realiza SIGMOIDECTOMIA. 3) En caso de HEMORRAGIA persistente se realiza COLECTOMIA parcial o total con ileostomía

con PROCEDIMIENTO DE HARTMAN.

Page 113: 4.GASTROENTEROLOGIA Y CIRUGIA

Diverticulitis aguda. Imágenes de TCMC axiales (A y B) y coronal (C) en un varón de 50 años con dolor en fosa ilíaca izquierda y fiebre: engrosamiento parietal de un segmento de colon descendente-sigma con reticulación de la grasa pericolónica y presencia de divertículos.

Page 114: 4.GASTROENTEROLOGIA Y CIRUGIA
Page 115: 4.GASTROENTEROLOGIA Y CIRUGIA
Page 116: 4.GASTROENTEROLOGIA Y CIRUGIA

POLIPOS DEL COLON• 111. Cual es la clasificación de los pólipos neoplásicos de colon?

R = A) TUBULARES 75% que corresponde a ADENOMAS. B) VELLOSOS 10% de los adenomas, mas relacionado con cáncer colonico y C) TUBULOVELLOSOS 15% de los adenomas. Requiriéndose un periodo mínimo de 10ª para que un adenoma se convierta a cáncer.

• 112. Cuales son las alteraciones genéticas implicadas en los adenomas y el cáncer de colon? R = Mutación del GEN RAS y lesiones de los cromosomas 5, 17 y 18.

• 113. Cual es el cuadro clínico de un paciente con SÍNDROME DE PEUTZ-JEGHERS? R = Es un TRASTORNO AD caracterizado POR MANCHAS HIPERPIGMENTADAS en los labios, mucosa bucal, cara

y dedos, PRESENTANDO HAMARTROMAS EN TODO EL TUBO DIGESTIVO e INTUSUCEPCION CON STDB.

• 114. Cual es el manejo del Síndrome de Peutz-Jeghers? R = Resección de pólipo sintomático por endoscopia

• 115. Cual es el cuadro clínico de la poliposis juvenil difusa? R = Es un TRASTORNO AD caracterizado por una población homogénea de pólipos, tanto hamartromas como

adenomas, los pólipos pueden ocasionar hemorragia e intususcepción en este síndrome. 10% RIESGO DE DESARROLLO DE CA DE COLON.

• 116. Cual es el manejo de la poliposis juvenil difusa? R = RESECCIÓN del colon con y anastomosis ileorectal. SEGUIMIENTO CON PROCTOSCOPIA CADA 6M y

extirpación de cualquier pólipo nuevo.

Page 117: 4.GASTROENTEROLOGIA Y CIRUGIA
Page 118: 4.GASTROENTEROLOGIA Y CIRUGIA

• 117. Cual es el cuadro clínico del SÍNDROME DE COWDEN?

1) Enfermedad AD caracterizada por HAMARTROMAS en todo el tubo digestivo, PAPULAS FACIALES Y BUCALES, CRECIMIENTOS QUERATOSICOS EN MANOS Y PIES.

2) Estos pacientes pueden presentar cáncer de mama, tiroides o de útero.

• 118. Cual es el cuadro clínico del SÍNDROME DE CRONKHITE-CANADA? R = NO ES HEREDITARIO y se caracteriza por HAMARTROMAS INTESTINALES generalizados, ALOPECIA,

PIGMENTACIÓN CUTÁNEA y ATROFIA DE NAILS. Los SÍNTOMAS INCLUYEN VOMITO, DIARREA, MALABSORCIÓN y enteropatía con perdida de proteínas, LA MAYORÍA DE LOS PACIENTES MUERE POCO DESPUÉS DEL DIAGNOSTICO.

• 119. Cual es el cuadro clínico de la poliposis adenomatosa familiar? R = TRASTORNO AD y su defecto se localiza en el CROMOSOMA 5. Se caracteriza por la presencia de mas de

100 POLIPOS en todo el colon y recto, 100% DESARROLLA CÁNCER.

• 120. Cual es el manejo de la poliposis adenomatosa familiar? R = Resección con RECTOCOLECTOMIA total con ileostomía.

• 121. Cual es el cuadro clínico de la ENFERMEDAD DE GARDNER?

R = POLIPOSIS ADENOMATOSA familiar que se asocia a OSTEOMIELITIS, QUISTES EPIDERMOIDES Y FIBROMAS DE LA PIEL.

• 122. Que es el SÍNDROME DE TURCOT? R = POLIPOSIS ADENOMATOSA FAMILIAR que se asocia con NEOPLASIAS DEL SNC.

• 123. Como se manifiesta un Adenocarcinoma de colon ubicado en el lado derecho? R = Perdida de peso, HEMATOQUZIA, anemia por déficit de hierro y UNA MASA ABDOMINAL EN EL CID.

• 124. Como se manifiesta un Adenocarcinoma de colon ubicado en el lado izquierdo? R = Perdida de peso, cambios en hábitos intestinales, RECTORRAGIA y dolor abdominal tipo cólico. Se debe

realizar una colonoscopia con biopsia.

Page 119: 4.GASTROENTEROLOGIA Y CIRUGIA
Page 120: 4.GASTROENTEROLOGIA Y CIRUGIA
Page 121: 4.GASTROENTEROLOGIA Y CIRUGIA
Page 122: 4.GASTROENTEROLOGIA Y CIRUGIA
Page 123: 4.GASTROENTEROLOGIA Y CIRUGIA
Page 124: 4.GASTROENTEROLOGIA Y CIRUGIA
Page 125: 4.GASTROENTEROLOGIA Y CIRUGIA
Page 126: 4.GASTROENTEROLOGIA Y CIRUGIA

CASO CLINICO

An 18-year-old woman was diagnosed 7 yearsearlier with precocious pseudopuberty secondaryto ovarian tumor. Physical examinationreveals oral and lingual dark pigmentation.Which of the following is the most likely diagnosis?

(A) Peutz-Jeghers syndrome(B) Gardner’s syndrome(C) Lynch syndrome(D) juvenile polyposis(E) Turcot’s syndrome

Page 127: 4.GASTROENTEROLOGIA Y CIRUGIA

(A) Intestinal polyposis is a possible indication

of Peutz-Jeghers syndrome associated with

dark brown spots on the lips and palate. There

is characteristic distribution of pigment around

lips, nose, eyes, and hands. Tumors of the ovary,

breast, pancreas, and endometrium are associated with this syndrome.

Page 128: 4.GASTROENTEROLOGIA Y CIRUGIA

CASO CLINICO

A 74-year-old man underwent some type ofpeptic ulcer surgery years ago. He has symptoms

that include abdominal pain and bloatingabout 30–40 minutes after eating, accompanied

by nausea. If he vomits, the symptoms arerelieved. Which of the following is the most

likely cause for his symptoms?

(A) early dumping syndrome(B) late dumping syndrome(C) bile reflux gastropathy

(D) retained gastric antrum(E) afferent loop syndrome

Page 129: 4.GASTROENTEROLOGIA Y CIRUGIA

(E) This pattern of symptoms is characteristic of

afferent loop syndrome. It is caused by distention

and incomplete drainage of the afferent

loop and requires surgical correction. Bacterial

overgrowth of the afferent loop is more

common. Its clinical presentation includes postprandial

abdominal pain, bloating, and diarrhea.

Fat and vitamin B12 malabsorption can

occur.

Page 130: 4.GASTROENTEROLOGIA Y CIRUGIA

ENFERMEDAD INFLAMATORIA INTESTINAL

• 125. Que entidades componen la enfermedad inflamatoria intestinal? R = CUCI, Chron y colitis indeterminada

• 126. Que es la CUCI? R = Patología inflamatoria LIMITADA A LA MUCOSA DEL COLON Y RECTO, la cual se extiende en forma

CONTINUA a partir del recto donde la PROCTOSIGMOIDITS ES LA MAS COMÚN.

• 127. Cual es el cuadro clínico de la CUCI? R = DIARREA SANGUINOLENTA, DOLOR abdominal DE TIPO CÓLICO, urgencia fecal, tenesmo, DISTENCIÓN y

meteorismo. El 85% de los px tienen una presentación LEVA A MODERADA, mientras que el RESTO muestra una evolución mas fulminante con el riesgo DE MEGACOLON TOXICO Y PERFORACIÓN.

• 128. Como diagnosticas CUCI? R = Colonoscopia o RECTOSIGMOIDOSCOPIA encontrando friabilidad y ulceración de la mucosa, se puede

encontrar TAMBIÉN PUS Y MOCO.

• 129. Como manejas CUCI? R = Los antinflamatorios de acción local derivados de la 5-ASA (SULFAZALASINA, MESALAMINA) son los

fármacos de primera línea en enfermedades leves a moderadas, disminuyen la inflamación al inhibir a la ciclooxigenasa y 5-lipooxigenasa de la mucosa intestinal. Los ESTEROIDES pueden ser utilizados. Suelen utilizarse INMUNOSUPRESORES como CICLOSPORINA, 6-MERCAPTOPURINA, METOTREXATE. Se utiliza EL METRONIDAZOL para disminuir la carga bacteriana. La indicación del tratamiento quirúrgico es cuando ha fallado el medico.

Page 131: 4.GASTROENTEROLOGIA Y CIRUGIA
Page 132: 4.GASTROENTEROLOGIA Y CIRUGIA
Page 133: 4.GASTROENTEROLOGIA Y CIRUGIA
Page 134: 4.GASTROENTEROLOGIA Y CIRUGIA

• 130. Que estructuras afecta la ENFERMEDAD DE CHRON?

R = Desde la BOCA HASTA EL ANO, PRINCIPALMENTE EN EL ILEON TERMINAL y en el 50% de los casos el recto no se encuentra afectado. La inflamación es de TIPO TRANSMURAL, colitis y ABSCESOS EN CRIPTAS con DISTRIBUCIÓN SEGMENTARIA.

• 131. Cual es un hallazgo patognomónico de enfermedad de Chron? R = LOS GRANULOMAS NO CASEIFICANTES que consisten en células gigantes multinucleadas

y epiteloides.

• 132. Como se visualiza a la endoscopia la enfermedad de Chron? R = Como ULCERAS SERPINTIGINOSAS profundas con aspecto de GIJARROS, salteados.

• 133. Cual es el cuadro clínico de la enfermedad de Chron? R = DOLOR abdominal TIPO CÓLICO principalmente EN FID por una ILEITIS terminal,

DIARREA casi siempre SIN SANGRE, FIEBRE Y FATIGA. La EC PERIANAL puede presentarse con dolor, tumefacción y DRENAJE DE FISTULAS O ABSCESOS. Cuenta con MANIFESTACIONES EXTRAINTESTINALES, entre las mas comunes se encuentran las ARTICULARES: Espondilitis anquilosante y artritis periférica. DERMATOLOGICAS: Eritema nodoso. OCULARES: Uveitis y conjuntivitis. BOCA: Estomatitis aftosa.

• 134. Cual es el manejo de la enfermedad de Chron? R = Derivados de la 5-ASA, en enfermedad grave sin respuesta ESTEROIDES. En caso de

presentación fistulizante o estenosante o en resistentes a esteroides se usa AZATRIOPINA O EL INFLIXIMAB. Quirúrgico cuando hay complicaciones.

Page 135: 4.GASTROENTEROLOGIA Y CIRUGIA
Page 136: 4.GASTROENTEROLOGIA Y CIRUGIA
Page 137: 4.GASTROENTEROLOGIA Y CIRUGIA
Page 138: 4.GASTROENTEROLOGIA Y CIRUGIA
Page 139: 4.GASTROENTEROLOGIA Y CIRUGIA

CASO CLINICO

A 35-year-old White man has a long past historyof diarrhea, rectal bleeding, crampyabdominal pain, and the passage of mucus.He now presents with a worsening of hissymptoms despite taking his medications. Abarium enema is performed and is shown inFig. 4–1. Which of the following is the mostlikely diagnosis of this new complication?

(A) toxic megacolon(B) amoebic colitis(C) appendicitis(D) ischemic colitis(E) annular carcinoma

Page 140: 4.GASTROENTEROLOGIA Y CIRUGIA

(E) The carcinoma has occurred in a patientwith ulcerative colitis. The barium enemashows a long, constricting lesion in the transversecolon, with the whole colon devoid ofhaustral

markings. Some pressure effects areseen in the ileum due to metastases. The diagnosisof ulcerative colitis is made from the clinicalsymptoms and proctosigmoidoscopicexamination of an abnormally inflamed colonicmucosa.

Page 141: 4.GASTROENTEROLOGIA Y CIRUGIA

CASO CLINICOA 28-year-old man presents with symptoms

of frequent bowel movements, crampy abdominalpain, and the passage of mucus. There is

no history of any bloody diarrhea, but recently,he developed joint discomfort in his hands,knees, and back. On examination he is thin,

and his abdomen is soft with voluntary guardingin the left lower quadrant. His joints are

not actively inflamed and there is an asymmetricdistribution. Which of the following is

the most likely diagnosis?

(A) lymphoma of the bowel(B) amyloid infiltration(C) chronic pancreatitis

(D) ulcerative colitis(E) tropical sprue

Page 142: 4.GASTROENTEROLOGIA Y CIRUGIA

(D) Joint involvement in inflammatory bowel

disease may involve sacroiliitis or specific large

joint peripheral arthritis. The latter type of

arthritis parallels the course of the bowel disease.

The sacroiliitis (spondylitic) variety follows

an independent course.

Page 143: 4.GASTROENTEROLOGIA Y CIRUGIA

CASO CLINICO

A 22-year-old man with inflammatory bowel

disease is noted to have a “string sign” in the

ileal area on barium enema. In which of the

following conditions is this sign most often

seen?

(A) in the stenotic or nonstenotic phase of

the disease

(B) in the stenotic phase only

(C) as a rigid, nondistensible phenomenon

(D) with gastric involvement

(E) with rectal involvement

Page 144: 4.GASTROENTEROLOGIA Y CIRUGIA

(A) The string sign represents long areas of circumferential

inflammation and fibrosis. It is

seen in the stenotic and nonstenotic phase of

Crohn’s disease. In addition to the string sign,

abnormal puddling of barium and fistulous

tracts are other helpful x-ray signs of ileitis.

Other radiologic findings in Crohn’s disease

include skip lesions, rectal sparing, small ulcerations,

and fistulas.

Page 145: 4.GASTROENTEROLOGIA Y CIRUGIA

MEGACOLON TOXICO

Page 146: 4.GASTROENTEROLOGIA Y CIRUGIA

OBSTRUCCION INTESTINAL

Page 147: 4.GASTROENTEROLOGIA Y CIRUGIA
Page 148: 4.GASTROENTEROLOGIA Y CIRUGIA
Page 149: 4.GASTROENTEROLOGIA Y CIRUGIA

VOLVULOS DEL COLON

Page 150: 4.GASTROENTEROLOGIA Y CIRUGIA
Page 151: 4.GASTROENTEROLOGIA Y CIRUGIA
Page 152: 4.GASTROENTEROLOGIA Y CIRUGIA
Page 153: 4.GASTROENTEROLOGIA Y CIRUGIA

ISQUEMIA MESENTERICA AGUDA• 135. Cual es la etiología de la isquemia mesentérica aguda? R = El origen principal deriva de trombos cardiacos en pacientes viejos con ANTECEDENTES DE ENFERMEDAD

CARDIACA (IAM, FA, estenosis mitral), afectando principalmente a LA ARTERIA MESENTÉRICA SUPERIOR O SUS RAMAS.

• 136. Cual es el cuadro clínico de la isquemia mesentérica aguda? R = VOMITO, DIARREA Y DOLOR ABDOMINAL CON SANGRADO de tubo digestivo. El abdomen se encuentra

blando CON RUIDOS PERISTÁLTICOS AUMENTADOS. Con la progresión de la isquemia el intestino SE DILATA, LA PERISTALSIS DISMINUYE, las permeabilidad de la pared intestinal se pierde liberando bacterias y pudiendo provocar un SHOCK SÉPTICO, FALLA CARDIACA Y/O MÚLTIPLE. Posteriormente el intestino se encuentra cianótico y edematoso pudiendo llegar a la NECROSIS ENTRE LAS 8-12 HRS DE INICIO, cuando la inflamación avanza hasta el peritoneo originando irritación hay peor pronostico.

• 137. Cuales son los estudios que pueden orientar el diagnostico de la isquemia mesentérica aguda? R = LAPAROTOMIA Y ARTERIOGRAFIA MESENTERICA es de elección. Citología hemática que indica

hemoconcentración y leucocitosis, GASOMETRIA con acidosis metabólica, CPK elevada. En las ÚLTIMAS ETAPAS de la isquemia se observa HIPERKALEMIA Y AZOEMIA. RX ABDOMEN con íleo adinámico con abdomen sin gases, aire extra peritoneal libre, en un inicio se puede encontrar el SIGNO DE LA HUELLA DIGITAL que indica edema de la pared intestinal, posteriormente NEUMATOSIS INTESTINAL (aire dentro de la pared intestinal y sistema venoso portal).

• 138. Cual es el manejo de la isquemia mesentérica aguda? R = Reanimación inicial con LÍQUIDOS si el px no esta en falla cardiaca y ANTI COAGULACIÓN sistémica con

sulfato de HEPARINA para evitar la propagación de los trombos, también es importante CORREGIR LA ACIDOSIS METABÓLICA. En px que presentan isquemia mesentérica aguda sin datos de infarto intestinal se pueden usar tratamientos NO QUIRÚRGICOS COMO LA TERAPIA TROMBOLITICA o infusión local de vasodilatadores transcateter.

Page 154: 4.GASTROENTEROLOGIA Y CIRUGIA

TROMBOSIS MESENTERICA

• 139. Que pacientes tienen el riesgo de desarrollo de trombosis venosa mesentérica?

R = Pacientes traumatizados, hipertensión portal, peritonitis o con hipercoagubilidad

• 140. Cual es el cuadro clínico de la trombosis venosa mesentérica? R = DOLOR ABDOMINAL DIFUSO, DISTENCIÓN ABDOMINAL, nauseas,

vómitos y signos de deshidratación

• 141. Cual es el estándar de oro para el dx de la trombosis venosa mesentérica?

R = TAC EN ESPIRAL con medio de contraste oral e IV

• 142. Cual es el manejo de la TVM? R = ANTI COAGULACIÓN E HIDRATACIÓN.

Page 155: 4.GASTROENTEROLOGIA Y CIRUGIA
Page 156: 4.GASTROENTEROLOGIA Y CIRUGIA
Page 157: 4.GASTROENTEROLOGIA Y CIRUGIA
Page 158: 4.GASTROENTEROLOGIA Y CIRUGIA

APENDICITIS AGUDA

Page 159: 4.GASTROENTEROLOGIA Y CIRUGIA

STDA

Page 160: 4.GASTROENTEROLOGIA Y CIRUGIA
Page 161: 4.GASTROENTEROLOGIA Y CIRUGIA
Page 162: 4.GASTROENTEROLOGIA Y CIRUGIA
Page 163: 4.GASTROENTEROLOGIA Y CIRUGIA
Page 164: 4.GASTROENTEROLOGIA Y CIRUGIA
Page 165: 4.GASTROENTEROLOGIA Y CIRUGIA
Page 166: 4.GASTROENTEROLOGIA Y CIRUGIA
Page 167: 4.GASTROENTEROLOGIA Y CIRUGIA

STDB

Page 168: 4.GASTROENTEROLOGIA Y CIRUGIA

MANEJO STDB

A. PACIENTES ESTABLES O CON SANGRADO CRÓNICO.Estudio diagnóstico electivo de acuerdo al siguiente esquema:1) - Historia, examen físico y exámenes de laboratorio ya descritos.2) - PANENDOSCOPÍA3) - Colonoscopía con ileoscopía retrógrada.4) - Radiología de doble contraste de intestino delgado y grueso.5) - Yeyunoscopía proximal con colonoscopio por vía oral.6) - Cintigrafía.7) - Angiografía.8) - Enteroscopía.9) - Endoscopía intraoperatoria.

Page 169: 4.GASTROENTEROLOGIA Y CIRUGIA

B. PACIENTES HEMODINÁMICAMENTE INESTABLES O CON SANGRADO MASIVO.Deben estudiarse hospitalizados, idealmente en U.C.I. durante la fase de estabilización

hemodinámica.

1. COLONOSCOPÍA DE URGENCIA: Debe intentarse AÚN SIN PREPARACIÓN, aunque a veces resulta impracticable.

2. Si el sangrado se ha detenido: COLONOSCOPÍA CON PREPARACIÓN adecuada.

3. Si la hemorragia persiste, luego de colonoscopía de urgencia, se realiza estudio cintigráfico. Si el estudio CINTIGRÁFICO ES POSITIVO, se deberá elegir de acuerdo a la condición del paciente, entre angiografía, colonoscopía de emergencia o, en casos extremos, la cirugía dirigida.

4. Infusión de VASOPRESINA. DETIENE LA HEMORRAGIA en un alto porcentaje de casos. Aunque su efecto transitorio, permite completar el estudio. SU EFECTO SOBRE LA MUSCULATURA INTESTINAL, PROVOCANDO EVACUACIÓN DEL REMANENTE DE SANGRE EN EL COLON, PUEDE INTERPRETARSE ERRÓNEAMENTE COMO PERSISTENCIA O ACENTUACIÓN DE LA HEMORRAGIA. Se ha utilizado también OCTEOTRIDE Y GLIPRESINA en infusión continua, con buenos resultados y sin los efectos colaterales de la vasopresina.

5. Embolización colectiva. Procedimiento de excepción, no exento de riesgos.

Page 170: 4.GASTROENTEROLOGIA Y CIRUGIA

C. TRATAMIENTO QUIRÚRGICO.

La cirugía es el único tratamiento definitivo de algunas causas de HDB. En raras ocasiones, en que el estudio completo es negativo, puede ser un recurso diagnóstico. Siempre es preferible la cirugía electiva, con preparación de colon, estabilidad hemodinámica y conocimiento del sitio de origen del sangrado.

En el ADULTO y especialmente en el MAYOR DE 50 AÑOS, las causas más frecuentes de sangrado agudo y masivo que pueden requerir cirugía son:

1) DIVERTICULOSIS COLÓNICA: En este caso la perfusión de VASOPRESINA POR VÍA ARTERIAL consigue detener la hemorragia de origen diverticular en el 90% DE LOS CASOS. El tratamiento quirúrgico debe reservarse para los pacientes con hemorragia persistente o con una alta tasa de recidivas en los que ha fallado el tratamiento angiográfico.

2) ANGIODISPLASIAS: La hemorragia por angiodisplasia suele ser intermitente y, debido a la frecuencia de lesiones difusas, puede producirse en distintas localizaciones del tubo digestivo. Por el mismo motivo, tras una hemicolectomía derecha indicada por hemorragia debida a angiodisplasia de ciego o colon derecho, puede aparecer recidiva hemorrágica por idénticas lesiones en otros puntos del tubo digestivo en un porcentaje que oscila entre el 5 y el 40% según las distintas series. Ello condiciona la actitud quirúrgica y hace aconsejable la resección de un segmento intestinal sólo en pacientes con hemorragias incoercibles o con una elevada tasa de recidivas tras haber fallado las medidas conservadoras. Éstas son la FOTOCOAGULACIÓN O LA ELECTROCOAGULACIÓN TRANSENDOSCÓPICAS, cuando las lesiones se han podido identificar por endoscopía, o la EMBOLIZACIÓN ARTERIAL CUANDO SE HA UTILIZADO LA ANGIOGRAFÍA PARA EL DIAGNÓSTICO.

3) PATOLOGÍA GRAVE: Combinada de ambos hemicolon (angiodisplasia y enfermedad diverticular grave de colon izquierdo).

Page 171: 4.GASTROENTEROLOGIA Y CIRUGIA

TIPS STDA Y STDB

o Los pacientes que requieran mas de 10u de paquetes globulares deberán recibir también PLASMA FRESCO CONGELADO, PLAQUETAS O AMBOS.

o La colocación de BANDAS ELÁSTICAS es el tratamiento de elección en la rotura de VARICES ESOFÁGICAS.

o El LAVADO NASO GÁSTRICO se utiliza para DIFERENCIAR entre el STDA Y STDB.

o La POSITIVIDAD EN EL ASPIRADO del tubo naso gástrico, RUIDOS INTESTINALES HIPERACTIVOS y elevación de los niveles del BUN apoyan el diagnostico de STDA.

o En la hematemesis secundaria a la ingesta de AINES se debe realizar una esofagogastroscopia y administrar un inhibidor de la bomba.

o El hematoma retroperitoneal produce una IMAGEN DE VIDRIO DESPULIDO en la radiografía de abdomen.

Page 172: 4.GASTROENTEROLOGIA Y CIRUGIA

ABSCESO ANAL

• 143. Cuales son las causas de absceso anorrectal especificas y no especificas?

R = ESPECIFICAS: Chron, CUCI, Tb, actinomicosis, ca, leucemias. NOESPECIFICAS: Infección bacteriana de glándulas anales

• 144. Cual es la complicación principal que sucede al drenar un absceso anal?

R = Posterior formación de FISTULAS

• 145. Cual es el cuadro clínico en un px que presenta un absceso anal? R = DOLOR ANAL INTENSO o una TUMORACIÓN ANAL MUY DOLOROSA que

apareció en forma reciente.

• 146. Cual es el manejo de un absceso perianal? R = DRENAJE Y METRONIDAZOL O CIPROFLOXACINO.

Page 173: 4.GASTROENTEROLOGIA Y CIRUGIA
Page 174: 4.GASTROENTEROLOGIA Y CIRUGIA
Page 175: 4.GASTROENTEROLOGIA Y CIRUGIA

TUMORES DEL CANAL ANAL

Page 176: 4.GASTROENTEROLOGIA Y CIRUGIA

TUMORES DEL ANO

Page 177: 4.GASTROENTEROLOGIA Y CIRUGIA

FISTULA ANAL

• 147. Cual es la etiología de una fistula anal? R = Abscesos crónicos, CHRON Y CUCI.

• 148. Cual es el cuadro clínico de las fistulas anales? R = Es característico la historia de un ABSCESO PREVIAMENTE DRENADO en la

misma localización, DOLOR PERIANAL, la principal molestia es la PERMANENTE SALIDA DE MATERIAL PURULENTO a través de un orificio que MANCHA LA ROPA INTERIOR.

• 149. Cual es el estudio de imagen utilizado para el diagnostico de las fistulas anales?

R = ULTRASONIDO RECTAL

• 150. Cual es el manejo de las fistulas anales? R = La forma para predecir por donde drena la fistula es la regla de GOODSALL-

SALMON y una vez identificada se realiza FISTULOTOMIA.

Page 178: 4.GASTROENTEROLOGIA Y CIRUGIA

FISURA ANAL

• 151. Cual es la etiología de las fisuras anales? R = ISQUEMIA EN MUCOSA POR ESFÍNTER HIPERTÓNICO.

• 152. Cual es el cuadro clínico de las fisuras anales? R = DOLOR intenso TIPO ARDOROSO que se relaciona al inicio

de la evacuación con una SENSACIÓN DE DESGARRO al pasar el bolo fecal. SANGRADO FRECUENTE, suele ser en POCA CANTIDAD.

• 153. Cual es el manejo de las fisuras anales? R = Baños de asiento, laxante, RELAJANTE DE MUSCULATURA

PÉLVICA COMO CREMA DE ISOSORBIDE, DILTIAZEM O NIFEDIPINO.

Page 179: 4.GASTROENTEROLOGIA Y CIRUGIA

HEMORROIDES

Page 180: 4.GASTROENTEROLOGIA Y CIRUGIA

HEMORROIDES

Page 181: 4.GASTROENTEROLOGIA Y CIRUGIA
Page 182: 4.GASTROENTEROLOGIA Y CIRUGIA

HERNIAS ABDOMINALES

• 154. De donde emerge la hernia inguinal directa? R = Emerge DENTRO DE LOS VASOS EPIGÁSTRICOS originándose en el piso del

conducto inguinal

• 155. De donde emerge la hernia inguinal indirecta? R = Por FUERA DE LOS VASOS EPIGÁSTRICOS y son DE ORIGEN CONGÉNITO siendo

mas FRECUENTES DEL LADO DERECHO.

• 156. De que manera se distingue a la ex fis una hernia inguinal directa de indirecta?

R = Una HERNIA INDIRECTA EJERCE CIERTA FUERZA CONTRA LA PUNTA DEL DEDO, en tanto que la DIRECTA PRESIONA LA PULPA DEL DEDO. Además cuando es colocado el dedo en el orificio inguinal y pedirle al paciente que tosa es posible parar la hernia indirecta y no así la directa.

• 157. Cual es la localización de la hernia crural? R = DEBAJO DEL LIGAMENTO INGUINAL, medial a los vasos femorales

manifestándose CON DOLOR EN LA REGIÓN INGUINAL, progresivo e incapacitante.

Page 183: 4.GASTROENTEROLOGIA Y CIRUGIA
Page 184: 4.GASTROENTEROLOGIA Y CIRUGIA

TIPOS DE HERNIAS

Page 185: 4.GASTROENTEROLOGIA Y CIRUGIA
Page 186: 4.GASTROENTEROLOGIA Y CIRUGIA

COLESTASIS

Page 187: 4.GASTROENTEROLOGIA Y CIRUGIA
Page 188: 4.GASTROENTEROLOGIA Y CIRUGIA
Page 189: 4.GASTROENTEROLOGIA Y CIRUGIA

ALTERACIONES DEL METABOLISMO DE LA BILIRRUBINA

Page 190: 4.GASTROENTEROLOGIA Y CIRUGIA
Page 191: 4.GASTROENTEROLOGIA Y CIRUGIA
Page 192: 4.GASTROENTEROLOGIA Y CIRUGIA
Page 193: 4.GASTROENTEROLOGIA Y CIRUGIA
Page 194: 4.GASTROENTEROLOGIA Y CIRUGIA

CASO CLINICOA 16-year old girl has recently been referred toyour family practice. She is a recent immigrant

from Southeast Asia, and has been taking isoniazid(INH) and rifampin for uncomplicated

tuberculosis. Routine blood tests are unremarkable,except for an elevated direct bilirubin.

Other liver enzymes and function tests are completelynormal. Which of the following is the

most likely diagnosis?

(A) hemolytic anemia(B) INH toxicity

(C) Crigler-Najjar syndrome type I(D) rifampin toxicity(E) Rotor’s syndrome

Page 195: 4.GASTROENTEROLOGIA Y CIRUGIA

(E) Rotor’s syndrome is one of the two rare

inherited disorders causing elevations in direct

bilirubin. The other is Dubin-Johnson syndrome,

and both have an excellent prognosis. Hemolytic

anemia causes elevation in indirect bilirubin,

and hematologic changes would be expected.

INH causes elevation in liver enzymes, and

although Rifampin can cause isolated hyperbilirubinemia,

it is of the indirect kind. Crigler-

Najjar type I is a severe disorder of neonates

with elevated indirect bilirubin.

Page 196: 4.GASTROENTEROLOGIA Y CIRUGIA

CASO CLINICO

A 29-year-old woman is found, on routineannual blood testing, to have an increase inunconjugated bilirubin. There is no evidenceof hemolysis and liver tests are otherwisenormal. Which of the following is the mostlikely diagnosis?

(A) Crigler-Najjar syndrome(B) Dubin-Johnson syndrome(C) Rotor’s syndrome(D) Gilbert’s syndrome(E) pregnanediol therapy

Page 197: 4.GASTROENTEROLOGIA Y CIRUGIA

(D) Gilbert’s syndrome may be associated with

impaired hepatic uptake of bilirubin. It is

caused by hereditary decrease in the activity

of glucoronosyltransferase in the uridine

diphosphate glycosyltransferase 1 (UGT1)

family. More severe enzyme deficits are the

cause of the two variants of Crigler-Najjar syndrome.

Page 198: 4.GASTROENTEROLOGIA Y CIRUGIA

HEPATITIS VIRAL

Page 199: 4.GASTROENTEROLOGIA Y CIRUGIA

VHA-PICORNAVIRUS

• 158. Cual es el dato de laboratorio que orienta al VHA?

R = IgM VS VHA AGUDO E IgG CUANDO YA HABÍA SIDO INFECTADO EL PACIENTE

• 159. Cual es el manejo del VHA? R = Sintomático, Inmunización pasiva como profilaxis postexposicion.

• 160. Cuales son las indicaciones de la VACUNA PASIVA del VHA? R = PERSONAS QUE PLANEAN UN VIAJE, contactos íntimos, PERSONAL DE

GUARDERÍAS, ASILOS E INSTITUCIONES. NO esta INDICADA en px que tienen DATOS CLÍNICOS DEL VHA

• 161. Cuales son las indicaciones de la VACUNA ACTIVA del VHA? R = ENFERMEDAD HEPÁTICA CRÓNICA, VARONES HOMOSEXUALES,

DROGADICTOS.

Page 200: 4.GASTROENTEROLOGIA Y CIRUGIA
Page 201: 4.GASTROENTEROLOGIA Y CIRUGIA

CASO CLINICO

A 24-year-old man develops acute onset ofanorexia, nausea, and vomiting. This is followed

by clinical jaundice and right upperquadrant discomfort 1 week later. Four weeksago, he was traveling in South America, wherehe ate food from street vendors. He also had

unprotected sex with a new partner whilethere. On examination, he is icteric with rightupper quadrant tenderness but the rest of theexamination is normal. His hepatitis serology is

positive for an agent that could have been preventedwith vaccination. (SELECT TWO)

Page 202: 4.GASTROENTEROLOGIA Y CIRUGIA

(A, E) Both hepatitis Aand E are usually spread

by the fecal-oral route. Other forms of transmission

are exceedingly uncommon.

Page 203: 4.GASTROENTEROLOGIA Y CIRUGIA

CASO CLINICO

Acouple returns from a trip to Central America

and develops nonspecific abdominal symptoms

followed by clinical jaundice. They stayed

at a resort, and ate fresh fruits and salads.

Serology for hepatitis virus spread by the fecaloral

route is positive

Page 204: 4.GASTROENTEROLOGIA Y CIRUGIA

(B, D) Only hepatitis B and D are frequently

spread by sexual transmission. It does not seem

to occur with hepatitis E, and the evidence is

equivocal for hepatitis Aand C.

Page 205: 4.GASTROENTEROLOGIA Y CIRUGIA

CASO CLINICO

A38-year-old woman is worried about a recent

potential exposure to an individual with active

viral hepatitis. She is seeking treatment for viral

hepatitis for which passive immunotherapy

can provide some protection

Page 206: 4.GASTROENTEROLOGIA Y CIRUGIA

(A, B) Immunoglobulin injection can provide

prophylaxis against hepatitis A, and it is felt

that hepatitis B immune globulin is protective

for hepatitis B.

Page 207: 4.GASTROENTEROLOGIA Y CIRUGIA

VHB- HEPADNAVIRIDAE• 162. Cual es el dato de laboratorio que te orienta a VHB?

1) HBs Ag, la persistencia de >6m sugiere infección crónica.

2) Anti HBs confiere inmunidad protectora ante el virus.

3) Hbe Ag es el antígeno que refiere la REPLICACIÓN ACTIVA del virus y la CONTAGIOSIDAD, en px con infección crónica permanece constante.

4) IgM antiHBc infección aguda.

• 163. Cual es el manejo del VHB?1) INTERFERON ALFA2) LAMIVUDINA (análogo de nucleosidos que inhibe la TRANSCRIPTASA REVERSA),

3) ADEFOVIR (análogo de nucleótidos sintético que inhibe a la DNA polimerasa del VHB),

4) ENTECAVIR ( análogo nucleosido deoxiguanina que inhibe la actividad del DNApolimerasa del VHB).

Page 208: 4.GASTROENTEROLOGIA Y CIRUGIA
Page 209: 4.GASTROENTEROLOGIA Y CIRUGIA
Page 210: 4.GASTROENTEROLOGIA Y CIRUGIA
Page 211: 4.GASTROENTEROLOGIA Y CIRUGIA

MARCADORES DE HEPATITIS

Page 212: 4.GASTROENTEROLOGIA Y CIRUGIA

CASO CLINICO

A 33-year-old man has never been vaccinatedfor hepatitis B. Serologic tests reveal negativehepatitis B surface antigen (HBsAg) and positiveantibody to surface antigen. Which of thefollowing conditions does this serologic patternbest fit with?

(A) previous hepatitis B infection(B) chronic active hepatitis(C) acute hepatitis B infection(D) poor prognosis(E) need for vaccine to hepatitis B

Page 213: 4.GASTROENTEROLOGIA Y CIRUGIA

(A) The antibody can be demonstrated in

80–90% of patients, usually late in convalescence,

and indicates relative or absolute immunity.

In contrast, HBsAg occurs very early and

disappears in <6 months. Persistence of HBsAg

indicates chronic infection. The pattern in this

patient is also seen postvaccination, and perhaps

as a consequence of remote infection.

Page 214: 4.GASTROENTEROLOGIA Y CIRUGIA

CASO CLINICO

A 24-year-old woman is pregnant with her first

baby at 14 weeks. She feels well and the pregnancy

is noncomplicated. Routine screening is

positive for chronic viral hepatitis for which

perinatal transmission is of major epidemiologic

significance. (SELECT ONE)

Page 215: 4.GASTROENTEROLOGIA Y CIRUGIA

(B) In neonates, the transmission of hepatitis B

results in a 90% probability of developing

chronic infection. The ongoing infection (often

resulting in hepatoma) is a major cause of morbidity

and mortality in many parts of the

world.

Page 216: 4.GASTROENTEROLOGIA Y CIRUGIA

VHC- FLAVIVIRIDAE

• 164. Cuales son los estudios de laboratorio recomendados para el diagnostico del VHC?

R = Detección del RNA mediante PCR. El ESTÁNDAR DE ORO ES LA BIOPSIA.

• 165. Cual es el manejo del VHC?1) INTERFERON ALFA ya sea solo o combinado con

RIVABIRINA por 6 a 12m. 2) EL mejor tratamiento es el TRANSPLANTE HEPÁTICO

en caso de pacientes con cirrosis y descompensados.

Page 217: 4.GASTROENTEROLOGIA Y CIRUGIA
Page 218: 4.GASTROENTEROLOGIA Y CIRUGIA
Page 219: 4.GASTROENTEROLOGIA Y CIRUGIA

CASO CLINICO

A 34-year-old man is seen for asymptomatic

elevations in his AST and ALT. He appears well

and the physical examination is normal. There

is no prior history of intravenous drug use,

blood transfusions, or multiple sex partners.

His hepatitis serology is positive for the virus,

most likely to lead to chronic infection.

(SELECT ONE)

Page 220: 4.GASTROENTEROLOGIA Y CIRUGIA

(C) Chronic hepatitis C infection occurs in

80–90% of patients. About 50–70% will have

evidence of chronic liver disease. In contrast,

only 1–10% of adults infected with hepatitis B

will go on to chronic infection.

Page 221: 4.GASTROENTEROLOGIA Y CIRUGIA

CASO CLINICO

A 45-year-old woman is evaluated for chronic

elevations in her liver enzymes (AST and ALT).

She feels well and her physical examination is

normal. Nonviral etiologies, such as medications

and autoimmune conditions, are ruled

out. Her serology is positive for the hepatitis

virus that can respond to specific antiviral therapy.

(SELECT TWO)

Page 222: 4.GASTROENTEROLOGIA Y CIRUGIA

(B, C) Interferon has been used in both hepatitis

B and C infection. The response is better in

hepatitis B infection. Combination therapy with

interferon and ribavirin is more effective in

hepatitis C.

Page 223: 4.GASTROENTEROLOGIA Y CIRUGIA

VHD• 166. Que caracteriza al VHD? R = Es EL VIRUS MAS PEQUEÑO con 36 nm, NECESITA DE LA ENVOLTURA DEL VHB para

sobrevivir y su medio de CONTAGIO ES EL MISMO QUE EL DEL VHB.

• 167. Cual es el curso del VHD en coinfección aguda con VHB? R = Tiende a ser LIMITADO Y ELIMINAR EL HBsAg POR LA RESPUESTA INMUNE, al mismo

tiempo que desaparece el VHD.

• 168. Cual es el curso del VHD en súperinfección con VHB? R = Exposición del VHD EN INDIVIDUO PREVIAMENTE INFECTADO POR VHB. LA

REPLICACIÓN DEL VHD ES MAYOR Y MÁS RÁPIDA al encontrar previamente expresado al

HBs Ag. En 70% de pacientes de los casos la infección se vuelve crónica y se encuentran mayores tasas de mortalidad

• 169. Cual es el estudio de laboratorio que orienta al diagnostico del VHD?

R = IgG anti VHD o por detección de RNA DE VHD EN SUERO.

• 170. Cual es el manejo del VHD? R = No hay tratamiento especifico, se ha encontrado mejoría con INTERFERON ALFA.

Page 224: 4.GASTROENTEROLOGIA Y CIRUGIA

CASO CLINICO

A23-year old sex trade worker develops acuteonsetjaundice and seeks medical attention.

There is no history of intravenous drug use ortravel. On examination, she is icteric, there is nolymphadenopathy, and the heart and lungs are

normal. Her liver span is 14 cm and the liveredge is smooth but tender. Her pelvic examination

is normal. Serology for viral hepatitistransmitted via the sexual route is positive.

(SELECT TWO)

Page 225: 4.GASTROENTEROLOGIA Y CIRUGIA

(B, D) Only hepatitis B and D are frequently

spread by sexual transmission. It does not seem

to occur with hepatitis E, and the evidence is

equivocal for hepatitis Aand C.

Page 226: 4.GASTROENTEROLOGIA Y CIRUGIA

CASO CLINICO

A 28-year-old man, who emigrated fromNorth Africa, is evaluated for chronic elevations

in liver enzymes (AST and ALT). He hasno symptoms of acute hepatitis, and his physical

examination is normal with no characteristicsof chronic liver disease. His hepatitis

serology is positive for a virus that requirespresence of another infectious agent beforebecoming clinically apparent. (SELECT ONE)

Page 227: 4.GASTROENTEROLOGIA Y CIRUGIA

(D) Hepatitis D is a defective ribonucleic acid

(RNA) virus that requires the helper function of

hepatitis B virus (or other hepadnavirus) for

its replication and expression.

Page 228: 4.GASTROENTEROLOGIA Y CIRUGIA

VHE- CALICIVIRUS

Page 229: 4.GASTROENTEROLOGIA Y CIRUGIA

FAMILIAS DE HEPATITIS VIRICA

• VHA

PICORNAVIRUS

• VHB

HEPADNAVIRUS

• VHC

FLAVIVIRUS

• VHE

CALICIVIRUS

Page 230: 4.GASTROENTEROLOGIA Y CIRUGIA

HEPATITIS AUTOINMUNE• 171. Que predisposición genética tienen las personas que padecen hepatitis autoinmune? R = HLA-B8, HLA-DR3 Y DR-52.

• 172. Que clasificaciones tiene la hepatitis autoinmune? TIPO I: Autoinmune clásica o Lupoide, se presenta en mujeres jóvenes donde 30% presenta trastornos como

TIROIDITIS AUTOINMUNE O CUCI. A diferencia de otros grupos se presenta HIPERGAMAGLOBULINEMIA EN SUERO 5-6 GR/DL.

TIPO II: Mas común en Europa y los anticuerpos que distinguen a este grupo son los ANTIMICROMOSOMA DE HÍGADO Y RINON TIPO 1 (ANTI LKM 1), asociada a enfermedades inmunológicas como el VITÍLIGO, DM 1, TIROIDITIS AUTOINMUNE.

TIPO III: Se encuentran anticuerpos anti antígeno hepático soluble/hígado- páncreas (anti-SLA/LP) encontrando mas frecuentemente la TIROIDITIS AUTOINMUNE COMO ASOCIADA.

• 173. Cual es el cuadro clínico de las hepatitis autoinmunes? R = MÚLTIPLES TELANGIECTASIAS, astenia, ARTRALGIAS, ESTRIAS CUTÁNEAS, acné, hirsutismo, AMENORREA Y

HEPATOMEGALIA.

• 174. Cuales son los CRITERIOS diagnósticos para hepatitis autoinmune?1) MAYORES: Elevación persistente de transaminasas, Hipergamaglobulinemia 2.5 o mayor, Ac circulantes y/o

células LE positivas, marcadores serológicos para virus de hepatitis negativos. 2) MENORES: Manifestaciones sistémicas como fiebre-artralgias-erupciones cutáneas, Complicaciones con

otras enfermedades autoinmunes , incluyendo las del colágeno, eritrosedimentacion elevada. 3) HISTOLOGIA: Hepatitis crónica o cirrosis con marcada infiltración de células y necrosis de hepatocitos,

ausencia de hallazgos específicos para el diagnóstico de otras entidades.

• 175. Cual es el manejo de la hepatitis autoinmune? R = De elección son los GLUCOCORTICOIDES SOLOS O COMBINADOS CON AZATRIOPINA. La remisión se

considera cuando los px demuestran mejoría clínica, niveles de transaminasas y gammaglobulinas normales en caso de recidivas se usa tratamiento inmunosupresor de por vida.

Page 231: 4.GASTROENTEROLOGIA Y CIRUGIA
Page 232: 4.GASTROENTEROLOGIA Y CIRUGIA

CAUSAS DE ELEVACION DEL AST O ALT

Page 233: 4.GASTROENTEROLOGIA Y CIRUGIA

HEPATITIS ALCOHOLICA

• 176. Cual es el cuadro clínico de un paciente con hepatitis alcohólica?

R = Perdida de peso, anorexia, ICTERICIA, FIEBRE Y HEPATOMEGALIA DOLOROSA.

• 177. Cuales son los hallazgos de laboratorio en la hepatitis alcohólica?

R = Aumento de las enzimas hepáticas, anemia, trombocitopenia y LEUCOCITOSIS CON PREDOMINIO DE NEUTROFILOS Y SE CONFIRMA EL DIAGNOSTICO POR BIOPSIA.

• 178. Cual es el manejo de la hepatitis alcohólica? R = Abstinencia, CORTICOIDES y plan nutricio.

Page 234: 4.GASTROENTEROLOGIA Y CIRUGIA

CASO CLINICO

A 45-year-old woman presents with a 1-week

history of jaundice, anorexia, and right upper

quadrant discomfort. On examination she is

icteric, with a tender right upper quadrant

and liver span of 14 cm. There is no shifting

dullness or pedal edema and the heart and

lungs are normal. On further inquiry, she

reports consuming one bottle of wine a day for

the past 6 months. Which of the following laboratory

tests is most likely to be characteristic

of a patient with jaundice secondary to alcoholic

hepatitis?

(A) ratio of AST:ALT is 3:1 and the AST is 500 U/L

(B) ratio of AST:ALT is 2:1 and the AST is 250 U/L

(C) ratio of AST:ALT is 1:1 and the AST is 500 U/L

(D) ratio of AST:ALT is 1:1 and the AST is 250 U/L

(E) ratio of AST:ALT is 1:3 and the AST is 750

Page 235: 4.GASTROENTEROLOGIA Y CIRUGIA

(B) In alcoholic hepatitis, the AST:ALT ratio is

usually >2, and the level of AST is usually <300.

When viral hepatitis or toxin-induced hepatitis

causes jaundice, the AST:ALT ratio is usually 1

or less, and the transaminases are usually >500.

Page 236: 4.GASTROENTEROLOGIA Y CIRUGIA

ABSCESO HEPATICO AMEBIANO

Page 237: 4.GASTROENTEROLOGIA Y CIRUGIA

ABSCESO HEPATICO AMEBIANO

Page 238: 4.GASTROENTEROLOGIA Y CIRUGIA

CASO CLINICO

Blood-filled lesions in the liver (peliosishepatis) are most likely to be seen with which

of the following medications?

(A) methyltestosterone(B) oral contraceptives

(C) trimethoprim-sulfamethoxazole(D) chlorpromazine

(E) erythromycin

Page 239: 4.GASTROENTEROLOGIA Y CIRUGIA

(A) Anabolic steroids usually cause cholestasis

and jaundice without inflammation. Sinusoidal

dilatation and peliosis occur less frequently,

and there have been deaths linked to peliosis.

The other medications also cause various types

of liver disease, but not peliosis.

Page 240: 4.GASTROENTEROLOGIA Y CIRUGIA

CIRROSIS HEPATICA• 179. Que parámetros definen la presión portal? R = Flujo venoso portal y la resistencia interior del hígado

• 180. Cuales son las causas mas frecuentes de cirrosis hepática?

R = Alcohol, HEPATITIS B Y C, METOTREXATO, alfametildopa e hidralazina.

• 181. Cual es la clasificación morfológica de la cirrosis?1) CIRROSIS MICRONODULAR: Nódulos menores de 3 mm. 2) CIRROSIS MACRONODULAR: Nódulos de mas de 3 mm. 3) CIRROSIS MIXTA: Mircro y macronodulares que ocurre comúnmente por

alcohol.

• 182. Cual es el marcador mas útil para el diagnostico de cirrosis biliar primaria?

R = ANTICUERPOS ANTIMITOCONDRIALES.

Page 241: 4.GASTROENTEROLOGIA Y CIRUGIA
Page 242: 4.GASTROENTEROLOGIA Y CIRUGIA

• 183. Cual es el cuadro clínico de la cirrosis? R = Asintomatico o los siguientes:A. SINTOMAS CONSTITUCIONALES: B. Astenia, anorexia, perdida de peso muscular.C. ASCITIS: D. Por retención de sodio y agua.E. SIGNOS CUTANEOS: F. Ictericia, palidez por la anemia. Telangiectacias cutáneas y eritema palmar (que predomina sobre las eminencias tenar e

hipotenar y en las llemas de los dedos) son datos bastantes caracteristicos.G. CIRCULACION COLATERAL: H. Los sitios mas comunes de aparición con significado patológico son el esófago y el fondo gástrico.I. ALIENTO: J. Olor dulzón debido a la exhalación de mercaptanos, productos derivados de la metionina por defecto de su metilación.K. ALTERACIONES ENDOCRINAS: L. Varones con signos de feminización como ginecomastia y cambios en la distribución del vello coroporal. El hipogonadismo en

varones produce atrofia testicular, perdida de la libido, impotencia. En ambos sexos el vello axilar es escaso. En las mujeres es frecuente la oligomenorrea, amenorrea y esterilidad.

M. ALTERACIONES HEMATOLOGICAS: N. Alteraciones en la coagulación dado que todos los factores se sintetizan en hígado menos el factor de von Willebrand.O. ALTERACIONES PULMONARES: P. Ascitis torácica o derrame pleuralQ. ALTERACIONES DIGESTIVAS: R. Hemorragia digestiva, colelitiasis.S. ALTERACIONES RENALES: T. Retención de sodio con ascitisU. SX HEPATORRENAL: V. Se trata de una insuficiencia renal funcional como respuesta a mecanismos de descompensación tales como el uso exagerado

de diuréticos, AINES, paracentesis. La histología es normal. Este síndrome se caracteriza por una insuficiencia renal funcional con función tubular normal en un px con insuficiencia hepática.

W. INFECCIONES: X. IVU mas comunes.Y. ALTERACIONES NEUROLOGICAS: Z. Encefalopatia hepática donde la primer manifestación en aparecer son los cambios en el comportamiento. El GABA es el

principal neurotransmisor inhibitorio del cerebro, se ha postulado que el GABA producido a nivel intestinal escapa del metabolismo hepático, atraviesa la barrera hematoencefalica y penetra al cerebro desencadenando la encefalopatía hepática.

Page 243: 4.GASTROENTEROLOGIA Y CIRUGIA

GRADOS DE ENCEFALOPATIA HEPATICA

Page 244: 4.GASTROENTEROLOGIA Y CIRUGIA
Page 245: 4.GASTROENTEROLOGIA Y CIRUGIA

• 184. Cual es el manejo de la cirrosis hepática? R = Dieta con contenido de proteínas 1 a 1.2 g/gr. Prohibicion del consumo de

alcohol. Hay tratamientos utiles específicos de cirrosis, como las flebotomías en la hemocromatosis, la d-penicilamina en la enfermedad de Wilson, los corticoides en las cirrosis autoinmunes, los antivíricos para cirrosis por VHB y VHC y el acido urodesoxicolico para la cirrosis biliar primaria. La cirugía se asocia a una mortalidad operatoria del 30% pero las indicaciones para la cirugía son rotura de varices o sangrados. A las varices tipo 2 y 3 se administra betabloqueador en caso que no exista contraindicación. Seguimiento ecográfico cada 6m para detectar hepatocarcinoma.

• 185. Cual es el manejo de emergencia en la cirrosis hepática? R = En caso de sangrado alto una endoscopia debe realizarse para dar

escleroterapia. Uso de vasopresina por via IV teniendo cuidado de la necrosis de la piel. Anastomosis portosistemica intrahepática por via transyugular TIPS para controlar la hemorragia digestica por varices esofágicas irreductible por otros métodos.

Page 246: 4.GASTROENTEROLOGIA Y CIRUGIA
Page 247: 4.GASTROENTEROLOGIA Y CIRUGIA
Page 248: 4.GASTROENTEROLOGIA Y CIRUGIA

PRONOSTICO CIRROSIS HEPATICA

• CLASE A 5-6: Pronóstico bueno,

• CLASE B 7-9: Pronóstico intermedio,

• CLASE C 10-15 PUNTOS: Pronóstico malo.

Page 249: 4.GASTROENTEROLOGIA Y CIRUGIA

CASO CLINICO

A 63-year-old man with a long history of alcoholabuse presents with ascites. He is experiencing

mild abdominal discomfort and nausea.Examination reveals tense ascites and generalized

tenderness but no rigidity. A diagnosticparacentesis of the fluid is performed. Which of

the following ascitic fluid results is most likelyto suggest an uncomplicated ascites due to

portal hypertension from cirrhosis?

(A) hemorrhage(B) protein >25 g/L

(C) bilirubin level twice that of serum(D) serum to ascites albumin gradienT >1.1 g/dL

(E) more than 1000 white cells/mm3

Page 250: 4.GASTROENTEROLOGIA Y CIRUGIA

(D) Ascitic fluid in uncomplicated cirrhosisof the liver shows a specific gravity <1.016.Protein is <25 g/L, and the gross appearance isstraw-colored. In spontaneous bacterial peritonitis,the fluid may be cloudy and the numberof white cells (neutrophils) increased. Inuncomplicated ascites, the difference betweenplasma albumin and ascitic fluid albumin is>1.1 g/dL.

Page 251: 4.GASTROENTEROLOGIA Y CIRUGIA

TUMORES BENIGNOS DE HIGADO

Page 252: 4.GASTROENTEROLOGIA Y CIRUGIA

HEPATOCARCINOMA

• 186. Cuales son los factores de riesgo para desarrollar un hepatocarcinoma? R = Aparece en personas que ya tienen alguna enfermedad hepática como HEPATITIS POR

VIRUS B O C, CIRROSIS.

• 187. Cual es el cuadro clínico de un px con hepatocarcinoma? R = Puede cursar asintomático. Cuando existen manifestaciones lo mas habitual es que se

presenten síntomas inespecíficos como baja de peso, dolor abdominal, saciedad precoz o masa palpable.

• 188. Cuales son los estudios de gabinete utilizados para diagnosticar hepatocarcinoma? R = TAC DINÁMICA HELICOIDAL MULTICORTE DE ELECCIÓN. Us de seguimiento. La

ALFAFETOPROTEINA es el marcador tumoral mas utilizado para el dx y seguimiento (puede elevarse en embarazo y tumores testiculares). Confirmación por biopsia.

• 189. Cual es el manejo del hepatocarcinoma? R = RESECCIÓN del tumor. INYECCIÓN DE ETANOL en tumor lo reseca y es fácil de realizar.

QUIMIOEMBOLIZACION que frecuentemente se complica con el llamado síndrome postembolizacion caracterizado por fiebre, elevación enzimática, dolor abdominal y nauseas. TRANSPLANTE HEPÁTICO.

Page 253: 4.GASTROENTEROLOGIA Y CIRUGIA

CASO CLINICO

A 55-year-old man from China is known to havechronic liver disease, secondary to hepatitis Binfection. He has recently felt unwell, and his

hemoglobin level has increased from 130 g/L, 1year ago, to 195 g/L. Which of the following is

the most appropriate initial diagnostic test?

(A) alkaline phosphatase(B) alpha-fetoprotein (AFP)

(C) aspartate transaminase (AST)(D) alanine transaminase (ALT)

(E) unconjugated bilirubin

Page 254: 4.GASTROENTEROLOGIA Y CIRUGIA

(B) Hepatoma is the most likely diagnosis inthis man. In China, it is estimated that the lifetimerisk of hepatoma in people with chronichepatitis B is close to 40%. AFP elevations, over500–1000 mg/L in the absence of a colonictumor (or pregnancy), suggest hepatoma.Paraneoplastic syndromes are not common butinclude erythrocytosis, hypercalcemia, andacquired porphyria.

Page 255: 4.GASTROENTEROLOGIA Y CIRUGIA

VESICULA BILIAR

• 190. Que fármacos pueden obliterar el esfínter de Oddi? R = Morfina y meperidina

• 191. Que elementos componen el síndrome de Mirizzi? R = COMPRESIÓN de los conductos biliares debida a INFLAMACIÓN intensa por colecistitis

secundaria a la IMPACTACIÓN de un lito en el infundíbulo de la vesícula.

• 192. Cual es el estándar de oro para el diagnostico de coledocolitiasis? R = CPRE

• 193. Cual es el cuadro clínico de la COLANGITIS? R = TRIADA DE CHARCOT que se compone de FIEBRE, DOLOR EN CUADRANTE DERECHO E

ICTERICIA. Cuando se le agregan cambios en el estado mental y choque se conoce como PENTADA DE REYNOLDS.

• 194. Cual es el estándar de oro para diagnostico de colangitis? R = CPRE. La presencia de leucocitosis, elevación de las bilirrubinas, FA y transaminasas

apoyan el diagnostico.

Page 256: 4.GASTROENTEROLOGIA Y CIRUGIA
Page 257: 4.GASTROENTEROLOGIA Y CIRUGIA
Page 258: 4.GASTROENTEROLOGIA Y CIRUGIA

CASO CLINICO

A 42-year-old previously well woman presentswith pruritus. She is not taking any medications,

and only drinks alcohol on a social basis.Her physical examination is entirely normal

with no signs of chronic liver disease or jaundice.Laboratory evaluation reveals an alkaline

phosphatase level of three times normal, andan ultrasound of the liver and gallbladder isnormal. Which of the following is the most

appropriate next step in diagnosis?

(A) INR or prothrombin time(B) antinuclear antibodies

(C) protein immunoelectrophoresis(D) abdominal ultrasound

(E) antimitochondrial antibodies

Page 259: 4.GASTROENTEROLOGIA Y CIRUGIA

(E) The patient with primary biliary cirrhosis

(PBC) is typically a middle-aged woman with

itching. Patients are often asymptomatic and

diagnosed only on routine blood work. The

cause of PBC is unknown, but a disordered

immune response may be involved. Apositive

antimitochondrial antibody test is found in over

90% of symptomatic patients.

Page 260: 4.GASTROENTEROLOGIA Y CIRUGIA

CASO CLINICO

A 33-year-old woman is recently diagnosedwith primary biliary cirrhosis (PBC). She is

inquiring about available treatments for thiscondition, including if there is a cure for the

disease. Which of the following treatments ismost likely to “cure” her PBC?

(A) ursodiol(B) methotrexate(C) azathioprine

(D) liver transplantation(E) glucocorticoids

Page 261: 4.GASTROENTEROLOGIA Y CIRUGIA

(D) There is no known effective therapy to preventprogression of liver disease in PBC. Inpatients with end-stage liver failure, livertransplantation is “curative,” since recurrenceafter transplantation is rare. The other therapieslisted have been reported as effective insmall case series but not in controlled trials.Ursodeoxycholic acid seems effective in providingat least symptomatic improvement, andmay even delay liver transplantation.Replacement of fat-soluble vitamins (e.g., vitaminA to prevent night blindness) is an importantpart of therapy, as is replacement ofcalcium and vitamin D to prevent osteoporosis.

Page 262: 4.GASTROENTEROLOGIA Y CIRUGIA

COLANGIOCARCINOMA

Page 263: 4.GASTROENTEROLOGIA Y CIRUGIA
Page 264: 4.GASTROENTEROLOGIA Y CIRUGIA

ANGIOSARCOMA

Page 265: 4.GASTROENTEROLOGIA Y CIRUGIA

HEMOCROMATOSIS• 195. Que tipo de trastorno es la hemocromatosis y que intercambio de aa se involucran?

• AR. Se produce por sustitución de CISTINA POR TIROSINA en la posición 282

(C282Y).

• Trastorno de almacenamiento de hierro en el que el incremento inapropiado de la absorción intestinal. Rara vez se presenta antes de los 20ª.

• 196. Cual es el cuadro clínico de la hemocromatosis? R = Los síntomas iniciales consisten en debilidad, cansancio, perdida de peso, cambio de

coloración de la piel, dolor abdominal, perdida de la libido y síntomas de DM. La

HEPATOMEGALIA 95% , HIPERPIGMENTACION 90 %(color gris pizarra o

metálico/ bronceado), POLIARTRITIS 20-50%, ICC 10%, angiomas en araña,

esplenomegalia, artropatía, ascitis, arritmias cardiacas, HIPOGONADISMO con perdida de vello, atrofia testicular y la ictericia son signos de enfermedad avanzada.

• 197. Como diagnosticas hemocromatosis? R = 1) Medición de hierro sérico y porcentaje de TRANSFERRINA > 50% EN AYUNO 2)

Medición de concentración de ferritina 3) Biopsia hepática

• 198. Cual es el manejo de la hemocromatosis? R = FLEBOTOMIAS de 500 ml una o 2 veces a la semana. DEFEROXAMINA cuando la anemia

o hipoproteinemia son muy graves.

Page 266: 4.GASTROENTEROLOGIA Y CIRUGIA
Page 267: 4.GASTROENTEROLOGIA Y CIRUGIA
Page 268: 4.GASTROENTEROLOGIA Y CIRUGIA

ENFERMEDAD DE WILSON• 199. Que causa la enfermedad de Wilson?

R = AR. Causado por mutaciones en el gen ATP 7B, cuyo producto es una ATPasa transportadora de cobre ligada a membrana.

• 200. Cual es el cuadro clínico de la enfermedad de Wilson? R = MANIFESTACIONES NEUROLÓGICAS manifestadas principalmente con trastornos del

movimiento como DISTONIA, INCOORDINACIÓN Y TEMBLORES. La disfagia y la disartria son comunes. Anillos de KAYSER-FLEISCHER.

• 201. Cuales son las pruebas diagnosticas para la enfermedad de Wilson?

R = Niveles séricos de CERULOPLASMINA BAJOS. Anillos de Kayser-Fleischer 99 %. Biopsia hepática con análisis cuantitativos de cobre con valores >3. 1mol/g.

• 202. Cual es el manejo de la enfermedad de Wilson?1) PENICILAMINA + PIRIDOXINA. 2) La TRIENTINA ha sustituido a la penicilamina. 3) ZINC es menos toxico.

• 203. Cual es el pronostico de la enfermedad de Wilson? R = BUENO, los síntomas neurológicos desaparecen una vez iniciado el tratamiento.

Page 269: 4.GASTROENTEROLOGIA Y CIRUGIA
Page 270: 4.GASTROENTEROLOGIA Y CIRUGIA
Page 271: 4.GASTROENTEROLOGIA Y CIRUGIA

CIRROSIS CARDIACA

Page 272: 4.GASTROENTEROLOGIA Y CIRUGIA

PANCREATITIS AGUDA• 204. Cual es el cuadro clínico de la pancreatitis aguda? R = Aparición súbita de un dolor en la región abdominal superior acompañado de nausea y vomito. El paciente

reporta el dolor en el epigastrio y en la región periumbilical, con irradiación hacia espalda, tórax, flancos y abdomen inferior. Tomando posición antialgica. A la EX FIS: Fiebre, hipotensión, rigidez abdominal, taquipnea y distención abdominal. Es posible encontrar los signos de CULLEN (hematoma periumbilical) o de GREY-TURNER (hematoma en los flancos) relacionados con hemorragias retroperitoneales.

• 205. Cuales son los hallazgos de laboratorio en la pancreatitis aguda?1) Elevación de la amilasa pancreática. 2) LIPASA PANCREÁTICA QUE SE ELEVA LAS PRIMERAS 4-8 HRS. 3) Niveles de AMILASA, TAC Y US TE ORIENTAN AL DIAGNOSTICO. En la pancreatitis aguda grave disminuye el

calcio.

• 206. Cual es el hallazgo radiológico sugestivo de pancreatitis aguda? R = ASA EN CENTINELA.

• 207. Cuales marcadores de gravedad de pancreatitis aguda hay? R = Balthazar, Ranson y APACHE II.

• 208. Cuales son los manejos de la pancreatitis aguda?1) TRATAMIENTO MEDICO: La pancreatitis leve se debe tratar con HIDRATACIÓN AGRESIVA hasta que el

paciente muestre balances hídricos positivos. El control del dolor se puede realizar con MORFINA. Nutrición enteral. En los casos graves, se recomienda ELIMINAR LA INGESTIÓN POR VIA ORAL, sin descuidar la nutrición y estado catabólico del paciente.

2) TX QUIRURGICO: En necrosis infectada

Page 273: 4.GASTROENTEROLOGIA Y CIRUGIA
Page 274: 4.GASTROENTEROLOGIA Y CIRUGIA
Page 275: 4.GASTROENTEROLOGIA Y CIRUGIA

• Los pacientes que presentan hasta dos de las caracteristicas previamentemencionadas tienen una mortalidad minima. Aquellos con tres a cinco de estos factores, presentaran una mortalidad 10 y 20% y los que tengan masde 5 presentan 50% de mortalidad.

Page 276: 4.GASTROENTEROLOGIA Y CIRUGIA
Page 277: 4.GASTROENTEROLOGIA Y CIRUGIA
Page 278: 4.GASTROENTEROLOGIA Y CIRUGIA

CASO CLINICO

A 77-year-old woman is brought to the emergency room because ofnonspecific abdominal discomfort. She has no anorexia, fever, chills,or weight loss. Her abdomen is soft and nontender on physicalexamination. Abdominal x-rays show lots of stool in the colon, butno free air or air-fluid levels. The amylase is 150 U/L (25–125 U/L),and the rest of her biochemistry and complete blood count arenormal. Which of the following conditions can cause a falsepositiveelevation in the serum amylase?

(A) maturity-onset diabetes mellitus (DM)(B) gastric ulcer(C) renal failure(D) sulfonamide therapy(E) gastric carcinoma

Page 279: 4.GASTROENTEROLOGIA Y CIRUGIA

(C) Amylase accumulates in the setting of renalfailure and thus becomes a less-valuable diagnostictest. Numerous other conditions involvingthe pancreas, the gut, and the salivaryglands can raise amylase levels. Sulfonamidescause pancreatitis; therefore, an elevated amylaseis not confusing, but rather a useful testfor pancreatitis in patients taking the drug.Morphine can elevate amylase levels in theabsence of pancreatitis.

Page 280: 4.GASTROENTEROLOGIA Y CIRUGIA

CASO CLINICO

A 29-year-old man with acquired immune deficiencysyndrome (AIDS) comes to the emergency

department because of progressively increasingabdominal discomfort. Examination shows voluntary

guarding in the upper abdomen. His biochemistryis normal except for an elevated

amylase at 370 U/L (25–125 U/L). Which of thefollowing infections can trigger this disorder in

AIDS patients?

(A) toxoplasmosis(B) Mycobacterium avium complex

(C) Mycobacterium tuberculosis(D) Pneumocystis carinii

(E) herpes virus

Page 281: 4.GASTROENTEROLOGIA Y CIRUGIA

(B) Pancreatitis in AIDS patients can be caused

by cytomegalovirus and cryptosporidium as

well as M. avium complex. Drugs are another

cause of AIDS-related pancreatitis.

Page 282: 4.GASTROENTEROLOGIA Y CIRUGIA

CASO CLINICO

A 53-year-old man presents to the emergencydepartment with severe epigastric abdominal

pain. His temperature is 37.2°C, blood pressure110/70 mm Hg, pulse 110/min, and respirations

20/min. Examination of the heart and lungsis normal, and his abdomen is tender in the epigastric

region. His white count is 15,000/mLand amylase is 450 U/L (25–125 U/L). Which ofthe following laboratory abnormalities is also

most likely to be present?

(A) hypoglycemia(B) hypercholesterolemia

(C) hyperglycemia(D) hypercalcemia

(E) hypercarbia

Page 283: 4.GASTROENTEROLOGIA Y CIRUGIA

(C) Hyperglycemia is very common in pancreatitis

and is usually multifactorial in origin.

Factors involved include decreased insulin

release, increased glucagon release, and elevated

adrenal glucocorticoids and catecholamines.

Page 284: 4.GASTROENTEROLOGIA Y CIRUGIA

PANCREATITIS CRONICA

• 209. Cuales son las causas de pancreatitis crónica? R = CONSUMO EXCESIVO DE ALCOHOL, enfermedades

autoinmunes, hipertrigliceridemia, hiperparatiroidismo, páncreas divisum y FIBROSIS QUÍSTICA.

• 210. Cual es el cuadro clínico de la pancreatitis crónica? R = ATAQUES INTERMITENTES de dolor intenso, de

localización a nivel abdominal superior con irradiación en forma de banda hacia la espalda.

• 211. Cuales son los estudios de imagen utilizados para el diagnostico de pancreatitis crónica?

R = TAC, US y CPRE.

Page 285: 4.GASTROENTEROLOGIA Y CIRUGIA
Page 286: 4.GASTROENTEROLOGIA Y CIRUGIA

CANCER DE PANCREAS

• 212. Cuales son los factores de riesgo para desarrollar cáncer de páncreas?

R = Tabaquismo, diabetes de larga evolución, pancreatitis crónica y NEM 1.

• 213. Cual es el cuadro clínico del cáncer de páncreas? R = Perdida de peso, fatiga, dolor abdominal epigástrico transfictivo,

ictericia, tromboflebitis migratoria, datos de hipertensión portal (hepatomegalia y ascitis) y VESÍCULA PALPABLE (signo de Courvoisier-Terrier).

• 214. Como diagnosticas cáncer de páncreas? R = TAC. Elevación FA y bilirrubina en sangre. MARCADOR TUMORAL CA 19-9.

• 215. Cual es el manejo del cáncer de páncreas? R = Quirúrgico. Quimioterapia.

Page 287: 4.GASTROENTEROLOGIA Y CIRUGIA

Tabla II: Síntomas y signos más frecuentes del cáncer de páncreas.

Síntomas-Anorexia, pérdida de peso, caquexia 50-90%-Dolor abdominal(epigastrio irradiado espalda) 40-90%-Ictericia progresiva 40-80%-Diabetes (sin antecedentes)(progresión de diabetes conocida) 20-85% -Síntomas depresivos 25-75% -Prurito (en ocasiones sin ictericia) 25-60% -Naúseas y vómitos 30-40%

Signos -Masa abdominal palpable (epigastrio) 25-30%-Fiebre (sin foco infeccioso conocido) 15-20% -Vesícula distendida no dolorosa (signo de Courvoisier) 10-15% -Edemas en extremidades inferiores 10-15%-Distensión abdominal 10-15%

Page 288: 4.GASTROENTEROLOGIA Y CIRUGIA
Page 289: 4.GASTROENTEROLOGIA Y CIRUGIA
Page 290: 4.GASTROENTEROLOGIA Y CIRUGIA

TRANSPLANTE HEPATICO

Page 291: 4.GASTROENTEROLOGIA Y CIRUGIA
Page 292: 4.GASTROENTEROLOGIA Y CIRUGIA
Page 293: 4.GASTROENTEROLOGIA Y CIRUGIA

COMPLICACIONES POSOPERATORIAS

• 216. Cuales son los diagnósticos diferenciales de la fiebre en el paciente quirúrgico?

1. Respuesta inflamatoria sistémica

2. ATELECTACIAS

3. Trombosis profunda

4. Flebitis

5. Alergias y fiebre medicamentosa

Las IVU son las mas frecuentes en los pacientes posoperados

El mayor beneficio del octreotide se encuentra en el cierre de fistulas pancreaticas.

Las fistulas enterocutaneas se clasifican en alto grado > 500 cc/24 hrs y bajo gasto <200 cc/24 hrs.

Page 294: 4.GASTROENTEROLOGIA Y CIRUGIA
Page 295: 4.GASTROENTEROLOGIA Y CIRUGIA
Page 296: 4.GASTROENTEROLOGIA Y CIRUGIA

CASO CLINICOA 67-year-old man is admitted to the intensive

care unit (ICU) because of respiratory failure

from pneumonia. He requires artificial ventilation

and inotropic support. On the third day in

the ICU, he develops melena and a drop in his

hemoglobin. Urgent upper endoscopy is performed

and it reveals three gastric ulcers in the

fundus and body of the stomach. Appropriate

treatment is started. (SELECT ONE)

(A) bile acid reflux implicated in pathogenesis

(B) immune or autoimmune pathogenesis suspected

(C) ischemia of the gastric mucosa implicated in the pathogenesis

(D) associated with Helicobacter pylori infection

(E) unknown mechanism

(F) interruption of prostaglandin synthesis

Page 297: 4.GASTROENTEROLOGIA Y CIRUGIA

(C) Acute erosive gastritis is most commonly

seen in critically ill hospitalized patients.

Ischemia of the gastric mucosa with breakdown

of the normal protective barriers of the stomach

is a key factor in the syndrome.

Page 298: 4.GASTROENTEROLOGIA Y CIRUGIA
Page 299: 4.GASTROENTEROLOGIA Y CIRUGIA

CICATRIZACION

Page 300: 4.GASTROENTEROLOGIA Y CIRUGIA

QUEMADURAS

Page 301: 4.GASTROENTEROLOGIA Y CIRUGIA
Page 302: 4.GASTROENTEROLOGIA Y CIRUGIA
Page 303: 4.GASTROENTEROLOGIA Y CIRUGIA
Page 304: 4.GASTROENTEROLOGIA Y CIRUGIA

MANEJO DEL PX POLITRAUMATIZADO

Page 305: 4.GASTROENTEROLOGIA Y CIRUGIA
Page 306: 4.GASTROENTEROLOGIA Y CIRUGIA

TRAUMATISMO TORAXICO

Page 307: 4.GASTROENTEROLOGIA Y CIRUGIA
Page 308: 4.GASTROENTEROLOGIA Y CIRUGIA

NEUMOTORAX Y HEMOTORAX TRAUMATICO

Page 309: 4.GASTROENTEROLOGIA Y CIRUGIA
Page 310: 4.GASTROENTEROLOGIA Y CIRUGIA

CONTUSION CARDIACA

Page 311: 4.GASTROENTEROLOGIA Y CIRUGIA

LESIONES DE LOS GRANDES VASOS

Page 312: 4.GASTROENTEROLOGIA Y CIRUGIA

TRAUMATISMOS ABDOMINALES

Page 313: 4.GASTROENTEROLOGIA Y CIRUGIA
Page 314: 4.GASTROENTEROLOGIA Y CIRUGIA
Page 315: 4.GASTROENTEROLOGIA Y CIRUGIA
Page 316: 4.GASTROENTEROLOGIA Y CIRUGIA
Page 317: 4.GASTROENTEROLOGIA Y CIRUGIA
Page 318: 4.GASTROENTEROLOGIA Y CIRUGIA
Page 319: 4.GASTROENTEROLOGIA Y CIRUGIA

TRAUMATISMOS DEL APARATO

GENITOURINARIO

Page 320: 4.GASTROENTEROLOGIA Y CIRUGIA
Page 321: 4.GASTROENTEROLOGIA Y CIRUGIA
Page 322: 4.GASTROENTEROLOGIA Y CIRUGIA
Page 323: 4.GASTROENTEROLOGIA Y CIRUGIA

LAPAROSCOPIA

Page 324: 4.GASTROENTEROLOGIA Y CIRUGIA
Page 325: 4.GASTROENTEROLOGIA Y CIRUGIA

SHOCK HIPOVOLÉMICO¿Cuándo un paciente está

en shock hipovolémico?

Pérdida estimada de sangre en un hombre de 70 Kg

CLASE I CLASE II CLASE III CLASE IV

Pérdida de

sangre (ml)< 750 750 - 1500 1500 - 2000 > 2000

Pérdida de

sangre (%)< 15 % 15 – 30 % 30 – 40 % > 40 %

Pulso < 100 > 100 > 120 > 140

Presión arterialNormal. Normal. Disminuida. Disminuida

Presión pulsoNormal Disminuida Disminuida Disminuida

Frecuencia

respiratoria14 - 20 20 - 30 30 - 40 > 35

Diuresis (ml/h)> 30 20 - 30 5 - 15 Mínima

Conciencia Levemente

ansioso

Moderada

ansioso.

Ansioso,

confuso.

Confuso,

letárgico.

Page 326: 4.GASTROENTEROLOGIA Y CIRUGIA
Page 327: 4.GASTROENTEROLOGIA Y CIRUGIA

PEDIATRIA

Page 328: 4.GASTROENTEROLOGIA Y CIRUGIA
Page 329: 4.GASTROENTEROLOGIA Y CIRUGIA

Consejos

1. Leer diariamente y en bloques

2. Adiós Partys un tiempo

3. Has ejercicio y come bien durante el estudio

4. Toma algún curso bueno si tienes la posibilidad

5. Ten Fe.

Page 330: 4.GASTROENTEROLOGIA Y CIRUGIA

BIBLIOGRAFIA

• EXARMED

• PAPADAKIS

• CTO

• HARRISON

• AMIR

• USMLE STEPS